You are on page 1of 96

MCQs PROMETRIC EXAM FOR NURSING

1. A pregnant client tells the clinic nurse that she wants to know the
sex of her baby as soon as it can be determined. The nurse
understands that the client should be able to find out at 12
weeks' gestation because by the end of the twelfth week the sex
of the fetus can be determined:
a.
b.
c.
d.

By the appearance of the external genitalia


Because the external genitalia begins to differentiate
Because the testes are descended into the scrotal sac
Because the internal differences in males and females
become apparent

2. A nursing instructor is reviewing the menstrual cycle with a


nursing student who will be conducting a prenatal teaching
session. The instructor asks the student to describe the folliclestimulating hormone (FSH) and the luteinizing hormone (LH). The
student accurately responds by stating that FSH and LH:
a.
b.
c.
d.

Are secreted by the adrenal glands


Are released from the anterior pituitary gland
Are secreted by the corpus luteum of the ovary
Stimulate the formation of milk during pregnancy

3. A nurse explains some of the purposes of the placenta to a client


during a prenatal visit. The nurse determines that the client
understands some of these purposes when the client states that
the placenta:
a.
b.
c.
d.

Cushions and protects the baby


Maintains the temperature of the baby
Is the way the baby gets food and oxygen
Prevents all antibodies and viruses from passing to the baby

4. A client arrives at the clinic for the first prenatal assessment. The
client tells a nurse that the first day of her last menstrual period
was October 19, 2012. Using Naegele's rule, the nurse determines
the estimated date of confinement is:
a.
b.
c.
d.

July 12, 2012


July 26, 2013
August 12, 2013
August 26, 2013

5. A pregnant client asks a nurse in the clinic when she will be able
to begin to feel the fetus move. The nurse responds by telling the
mother that fetal movements will be noted between which of the
following weeks of gestation?
a.
b.
c.
d.

6 and 8
8 and 10
10 and 12
16 and 20

6. A nurse is providing instructions to a client in the first trimester of


pregnancy regarding measures to assist in reducing breast
tenderness. The nurse tells the client to:
a.
b.
c.
d.

Avoid wearing a bra.


Wash the breasts with warm water and keep them dry.
Wear tight-fitting blouses or dresses to provide support.
Wash the nipples and areolar area daily with soap, and
massage the breasts with lotion.

7. A nurse is describing cardiovascular system changes that occur


during pregnancy to a client and understands that which finding
would be normal for a client in the second trimester?
a.
b.
c.
d.

Increase in pulse rate


Increase in blood pressure
Frequent bowel elimination
Decrease in red blood cell production

8. A nurse is performing an initial assessment on a client who has


just been told that a pregnancy test is positive. Which assessment
finding would indicate that the client is at risk for preterm labor?
a.
b.
c.
d.

The client is a 35-year-old primigravida.


The client has a history of cardiac disease.
The client's hemoglobin level is 13.5 g/dL.
The client is a 20-year-old primigravida of average weight
and height.

9. A nurse is performing an assessment on a pregnant client at 16


weeks of gestation. On assessment, the nurse expects the fundus
of the uterus to be located at which of the following areas?
a. Midway between the symphysis pubis and the
umbilicus
b. At the umbilicus
c. Just above the symphysis pubis
d. At the level of the xiphoid process

10.
A nurse is instructing a pregnant client regarding measures to
increase iron in the diet. The nurse tells the client to consume
which food that contains the highest source of dietary iron?
a.
b.
c.
d.

Milk
Potatoes
Cantaloupe
Dark green leafy vegetables

11.
A pregnant adolescent client asks the nurse about the
menstrual cycle. The nurse describes the cycle and tells the
adolescent that its normal duration is about how many days?
a.
b.
c.
d.

14
28
30
45

12.
A nurse is reviewing the record of a pregnant client seen in
the health care clinic for the first prenatal visit. Which of the
following data, if noted on the client's record, would alert the
nurse that the client is at risk for a spontaneous abortion?
a.
b.
c.
d.

Age of 35 years
History of syphilis
History of genital herpes
History of diabetes mellitus

13.
A pregnant client has been diagnosed with a vaginal infection
from the organism Candida albicans. Which of the following
findings would the nurse expect to note on assessment of the
client?
a.
b.
c.
d.

Absence of any signs and symptoms


Pain, itching, and vaginal discharge
Proteinuria, hematuria, edema, and hypertension
Costovertebral angle pain

14.
A client with myocardial infarction suddenly becomes
tachycardic, shows signs of air hunger, and begins coughing
frothy, pink-tinged sputum. Which of the following would the
nurse anticipate when auscultating the client's breath sounds?
a.
b.
c.
d.

Stridor
Crackles
Scattered rhonchi
Diminished breath sounds

15.
A client is having frequent premature ventricular contractions.
A nurse would place priority on assessment of which of the
following?
a.
b.
c.
d.

Sensation of palpitations
Causative factors, such as caffeine
Precipitating factors, such as infection
Blood pressure and oxygen saturation

16.
A nurse is assessing the neurovascular status of a client who
returned to the surgical nursing unit 4 hours ago after undergoing
aortoiliac bypass graft. The affected leg is warm, and the nurse
notes redness and edema. The pedal pulse is palpable and
unchanged from admission. How would the nurse correctly
interpret the client's neurovascular status?
a. The neurovascular status is normal because of
increased blood flow through the leg.
b. The neurovascular status is moderately impaired, and the
surgeon should be called.
c. The neurovascular status is slightly deteriorating and should
be monitored for another hour.
d. The neurovascular status is adequate from an arterial
approach, but venous complications are arising.
17.
A client with angina complains that the anginal pain is
prolonged and severe and occurs at the same time each day,
most often at rest in the absence of precipitating factors. How
would the nurse best describe this type of anginal pain?
a.
b.
c.
d.

Stable angina
Variant angina
Unstable angina
Nonanginal pain

18.
A clinic nurse is providing instructions to a client with
hypertension who will be taking captopril (Capoten). Which
statement by the client indicates a need for further instruction?
a.
b.
c.
d.

"I need to drink increased amounts of water."


"I need to change positions slowly."
"I need to avoid taking hot baths or showers."
"I need to sit down and rest if dizziness or lightheadedness
occurs."

19.
A nurse is developing a plan of care for a client with varicose
veins in whom skin breakdown occurred over the varicosities as a
result of secondary infection. A priority intervention in the plan of
care is to:
a.
b.
c.
d.

Keep the legs aligned with the heart.


Position the client onto the side every shift.
Clean the skin with alcohol every hour.
Elevate the legs higher than the heart.

20.
The clinic nurse is reviewing the assessment findings for a
client who has been taking spironolactone (Aldactone) for
treatment of hypertension. Which of the following, if noted in the
client's record, would indicate that the client is experiencing a
side effect related to the medication?
a.
b.
c.
d.

A potassium level of 3.2 mEq/L


A potassium level of 5.8 mEq/L
Client complaint of constipation
Client complaint of dry skin

21.
A nurse is caring for a client who has been hospitalized with a
diagnosis of angina pectoris. The client is receiving oxygen via
nasal cannula at 2 L/min. The client asks why the oxygen is
necessary. The nurse accurately explains that:
a. Oxygen has a calming effect.
b. Oxygen will prevent the development of any thrombus.
c. Oxygen dilates the blood vessels so they can supply more
nutrients to the heart muscle.
d. The pain of angina pectoris occurs because of a
decreased oxygen supply to heart cells.
22.
A nurse has an order to remove the nasogastric (NG) tube
from a client on the first postoperative day after cardiac surgery.
The nurse would question the prescription if which of the following
was noted on assessment of the client?
a.
b.
c.
d.

Bowel sounds are absent.


The abdomen is slightly distended.
NG tube drainage is Hematest negative.
The client is drowsy.

23.
A home health nurse is visiting an older client whose family
has gone out for the day. During the visit, the client experiences
chest pain that is unrelieved by three sublingual nitroglycerin

tablets given by the nurse. Which action by the nurse would be


appropriate at this time?
a. Notify a family member who is the next of kin.
b. Inform the home care agency supervisor that the visit may
be prolonged.
c. Call for an ambulance to transport the client to the
hospital emergency department.
d. Drive the client to the physician's office.
24.
A client has experienced a myocardial infarction. The nurse
plans care for the client knowing that the client's chest pain is due
to tissue hypoxia in which of the following layers of the heart?
a.
b.
c.
d.

Parietal pericardium
Visceral pericardium
Myocardium
Endocardium

25.
A nurse is assessing an electrocardiogram (ECG) rhythm strip
for a client. The P waves and QRS complexes are regular. The PR
interval is 0.14 second, and the QRS complexes measure 0.08
second. The overall heart rate is 82 beats/min. The nurse
interprets the cardiac rhythm to be:
a.
b.
c.
d.

Sinus bradycardia
Sick sinus syndrome
Normal sinus rhythm
First-degree heart block

26.
A nurse is teaching a client with cardiomyopathy about home
care safety measures. The nurse addresses which of the following
as the most important measure to ensure client safety?
a.
b.
c.
d.

Assessing pain
Avoiding over-the-counter medications
Administering vasodilators
Moving slowly from a sitting to a standing position

27.
A client receiving parenteral nutrition (PN) complains of a
headache. A nurse notes that the client has an increased blood
pressure, bounding pulse, jugular vein distention, and crackles
bilaterally. The nurse determines that the client is experiencing
which complication of PN therapy?
a. Sepsis
b. Air embolism
c. Hypervolemia

d. Hyperglycemia
28.
A nurse on the day shift walks into a client's room and finds
the client unresponsive. The client is not breathing and does not
have a pulse, and the nurse immediately calls out for help. The
next nursing action is which of the following?
a.
b.
c.
d.

Open the airway.


Give the client oxygen.
Start chest compressions.
Ventilate with a mouth-to-mask device.

29.
A nurse is performing cardiopulmonary resuscitation (CPR) on
a 7-year-old child. The nurse delivers how many breaths per
minute to the child?
a.
b.
c.
d.

6
8
10
20

30.
A nurse caring for a client with a chest tube turns the client to
the side and the chest tube accidentally disconnects. The initial
nursing action is to:
a.
b.
c.
d.

Call the physician.


Place the tube in a bottle of sterile water.
Immediately replace the chest tube system.
Place a sterile dressing over the disconnection site.

31.
The nurse is administering fluids intravenously as prescribed
to a client who sustained superficial partial-thickness burn injuries
of the back and legs. In evaluating the adequacy of fluid
resuscitation, the nurse understands that which of the following
would provide the most reliable indicator for determining the
adequacy?
a.
b.
c.
d.

Vital signs
Urine output
Mental status
Peripheral pulses

32.
A nurse is preparing to administer a dose of naloxone
hydrochloride (Narcan) intravenously to a client with an
intravenous opioid overdose. Which supportive medical
equipment would the nurse plan to have at the client's bedside if
needed?

a.
b.
c.
d.

Nasogastric tube
Paracentesis tray
Resuscitation equipment
Central line insertion tray

33.
A client who had cardiac surgery 24 hours ago has a urine
output averaging 20 mL/hr for 2 hours. The client received a
single bolus of 500 mL of intravenous fluid. Urine output for the
subsequent hour was 25 mL. Daily laboratory results indicate that
the blood urea nitrogen level is 45 mg/dL and the serum
creatinine level is 2.2 mg/dL. Based on these findings, the nurse
would anticipate that the client is at risk for which of the
following?
a.
b.
c.
d.

Hypovolemia
Acute renal failure
Glomerulonephritis
Urinary tract infection

34.
A client in ventricular fibrillation is about to be defibrillated. A
nurse knows that to convert this rhythm effectively, the biphasic
defibrillator should be set at which of the following energy levels
(in joules, J)?
a.
b.
c.
d.

50 J
80 J
200 J
360 J

35.
A community health nurse is providing a teaching session to
firefighters in a small community regarding care of a burn victim
at the scene of injury. The nurse instructs the firefighters that in
the event of a tar burn, the immediate action should be:
a.
b.
c.
d.

Cooling the injury with water


Removing all clothing immediately
Removing the tar from the burn injury
Leaving any clothing that is saturated with tar in place

36.
A nurse has developed a nursing diagnosis of Ineffective
airway clearance for a client who sustained an inhalation burn
injury. Which of the following nursing interventions should the
nurse include in the plan of care for this client?
a. Monitor oxygen saturation levels every 4 hours.
b. Encourage coughing and deep breathing every 4 hours.

c. Elevate the head of the bed.


d. Assess respiratory rate and breath sounds every 4 hours.
37.
A nurse is caring for a client who sustained a thermal burn
caused by the inhalation of steam 24 hours ago. The nurse
determines that the priority nursing action is to assess which of
the following?
a.
b.
c.
d.

Ability to swallow
Pain level
Lung sounds
Laboratory results

38.
A client is brought into the emergency department in
ventricular fibrillation (VF). The advanced cardiac life support
(ACLS) nurse prepares to defibrillate by placing conductive gel
pads on which part of the chest?
a. The upper and lower halves of the sternum
b. The right of the sternum, just below the clavicle and
to the left of the precordium
c. The right shoulder and the back of the left shoulder
d. Parallel between the umbilicus and the right nipple
39.
A nurse receives a telephone call from a neighbor, who states
that her 3-year-old child was found sitting on the kitchen floor
with an empty bottle of liquid furniture polish. The mother of the
child tells the nurse that the bottle was half full, that the child's
breath smells like the polish, and that spilled polish is present on
the front of the child's shirt. The nurse tells the mother to:
a. Call the poison control center.
b. Wait until the nurse comes to bring the child to the
emergency department.
c. Call the pediatrician.
d. Induce vomiting immediately.
40.
Which of the following should the nurse do when setting up an
arterial line?
a.
b.
c.
d.

Tighten all tubing connections.


Use macrodrop intravenous tubing.
Level the transducer to the ventricle.
Raise the height of the normal saline infusion to prevent
backup.

41.
When communicating with a client who speaks a different
language, the best practice for a nurse is to:
a. Speak loudly and slowly.
b. Stand close to the client and speak loudly.
c. Arrange for an interpreter when communicating with
the client.
d. Speak to the client and family together to increase the
chances that the topic will be understood.
42.
An antihypertensive medication has been prescribed for a
client with hypertension. The client tells a clinic nurse that she
would like to take an herbal substance to help lower her blood
pressure. The nurse should take which appropriate action?
a. Tell the client that herbal substances are not safe and should
never be used.
b. Encourage the client to discuss the use of an herbal
substance with the physician.
c. Teach the client how to take her blood pressure so that it can
be monitored closely.
d. Tell the client that if she takes the herbal substance she will
need to have her blood pressure checked frequently.
43.
The nurse understands that which of the following statements
regarding herbal and health therapies is correct?
a.
b.
c.
d.

Zinc is used for insomnia.


Ginger is used to improve memory.
Echinacea is used for erectile dysfunction.
Black cohosh produces estrogen-like effects.

44.
A nursing student is discussing cultural diversity issues in a
clinical conference when the nursing instructor asks the student
to describe ethnocentrism. Which of the following statements, if
made by the student, would indicate a lack of understanding of
the concept of ethnocentrism?
a. "It is a tendency to view one's own ways as best."
b. "It is acting in a superior manner toward other cultures."
c. "It is believing that one's own way is the only acceptable
way."
d. "It is imposing one's beliefs on individuals from
another culture."

45.
A nurse is implementing the complementary therapy of
therapeutic touch when caring for clients. When performing
therapeutic touch, the nurse should:
a.
b.
c.
d.

Apply heating pads to the back.


Vigorously massage bony prominences.
Position hands 2 to 4 inches from the body.
Position hands directly on the client's skin.

46.
A nurse is assigned to care for four clients. In planning client
rounds, which client should the nurse assess first?
a.
b.
c.
d.

A client scheduled for a chest x-ray


A client requiring daily dressing changes
A postoperative client preparing for discharge
A client receiving nasal oxygen who had difficulty
breathing during the previous shift

47.
A registered nurse is planning the client assignments for the
day. Which of the following is the most appropriate assignment for
a nursing assistant?
a.
b.
c.
d.

A client requiring a colostomy irrigation


A client receiving continuous tube feedings
A client who requires urine specimen collections
A client with difficulty swallowing food and fluids

48.
The nurse determines that a client is having a transfusion
reaction. After the nurse stops the transfusion, which action
should immediately be taken next?
a.
b.
c.
d.

Remove the intravenous (IV) line.


Run a solution of 5% dextrose in water.
Run normal saline at a keep-vein-open rate.
Obtain a culture of the tip of the catheter device removed
from the client.

49.
The nurse manager is observing a new nursing graduate
caring for a burn client in protective isolation. The nurse manager
intervenes if the new nursing graduate planned to implement
which incorrect component of protective isolation technique?
a. Using sterile sheets and linens
b. Performing strict hand-washing technique
c. Wearing gloves and a gown only when giving direct
care to the client
d. Wearing protective garb, including a mask, gloves, cap, shoe
covers, gowns, and plastic apron

50.
The police arrive at the emergency department with a client
who has seriously lacerated both wrists. The initial nursing action
is to:
a.
b.
c.
d.

Administer an antianxiety agent.


Examine and treat the wound sites.
Secure and record a detailed history.
Encourage and assist the client to ventilate feelings.

51.
A community health nurse is working with disaster relief after
a tornado. The nurse's goal for the community is to prevent as
much injury and death as possible from the uncontrollable event.
Finding safe housing for survivors, providing support to families,
organizing counseling, and securing physical care when needed
are all examples of which type of prevention?
a. Primary level of prevention
b. Secondary level of prevention
c. Tertiary level of prevention
d. Aggregate care prevention
52.
A nurse educator presents an in-service training session on
case management to nurses on the clinical unit. During the
presentation the nurse educator clarifies that a characteristic of
case management is that it:
a. Requires a case manager who plans the care for all of the
clients
b. Requires that one nurse take care of one client
c. Uses a team approach but one nurse supervises all of the
other employees
d. Promotes appropriate use of hospital personnel
53.
The emergency department nurse is caring for a child with
suspected epiglottitis and has ensured that the child has a patent
airway. The next priority in the care of this child would be which of
the following?
a.
b.
c.
d.

Prepare the child for a chest radiograph.


Assist the physician with intubation.
Prepare the child for tracheotomy.
Prepare to administer epinephrine.

54.
The nurse would instruct the nursing assistant to use an
electric razor for which of the following clients?
a. The postoperative client

b. The client taking acetaminophen (Tylenol)


c. The client taking warfarin (Coumadin)
d. The client with an infection
55.
A nurse is caring for a client who has just returned from
having a right-sided renal biopsy. The nurse should intervene if a
nursing assistant is observed doing which of the following?
a.
b.
c.
d.

Obtaining the client's vital signs


Positioning the client on the left side
Positioning the client on the right side
Providing the client with reading materials

56.
A clinic nurse is preparing to discuss the concepts of moral
development with a mother. The nurse understands that
according to Kohlberg's theory of moral development, in the preconventional level, moral development is thought to be motivated
by which of the following?
a.
b.
c.
d.

Peer pressure
Social pressures
Parents' behavior
Punishment and reward

57.
The mother of an 8-year-old child tells the clinic nurse that she
is concerned about the child because the child seems to be more
attentive to friends than anything else. Using Erikson's
psychosocial development theory, the appropriate nursing
response is which of the following?
a. "You need to be concerned."
b. "You need to monitor the child's behavior closely."
c. "At this age, the child is developing his own
personality."
d. "You need to provide more praise to the child to stop this
behavior."
58.
A nursing instructor asks a nursing student to present a
clinical conference to peers regarding Freud's psychosexual
stages of development, specifically the anal stage. The student
plans the conference, knowing that which of the following most
appropriately relates to this stage of development?
a. This stage is associated with toilet training.
b. This stage is characterized by the gratification of self.
c. This stage is characterized by a tapering off of conscious
biological and sexual urges.

d. This stage is associated with pleasurable and conflicting


feelings about the genital organs.
59.
A nurse is monitoring a 3-month-old infant for signs of
increased intracranial pressure. On palpation of the fontanels, the
nurse notes that the anterior fontanel is soft and flat. Based on
this finding, which nursing action is appropriate?
a.
b.
c.
d.

Increase oral fluids.


Notify the physician.
Document the finding.
Elevate the head of the bed to 90 degrees.

60.
A nurse is evaluating the developmental level of a 2-year-old.
Which of the following does the nurse expect to observe in this
child?
a.
b.
c.
d.

Uses a fork to eat


Uses a cup to drink
Pours own milk into a cup
Uses a knife for cutting food

61.
A mother arrives at a clinic with her toddler and tells a nurse
that she has a difficult time getting the child to go to bed at night.
Which of the following is appropriate for the nurse to suggest to
the mother?
a.
b.
c.
d.

Avoid a nap during the day.


Allow the child to set bedtime limits.
Allow the child to have temper tantrums.
Inform the child of bedtime a few minutes before it is
time for bed.

62.
The mother of a 3-year-old is concerned because her child still
is insisting on a bottle at nap time and at bedtime. Which of the
following is the appropriate suggestion to the mother?
a.
b.
c.
d.

Allow the bottle if it contains juice.


Allow the bottle if it contains water.
Do not allow the child to have the bottle.
Allow the bottle during naps but not at bedtime.

63.
The nurse is teaching an older client about measures to
prevent constipation. Which statement, if made by the client,
indicates that further teaching about bowel elimination is
necessary?
a. "I walk 1 to 2 miles every day."

b. "I need to decrease fiber in my diet."


c. "I have a bowel movement every other day."
d. "I drink six to eight glasses of water every day."
64.
The mother of a 16-year-old tells a nurse that she is
concerned because her child sleeps about 8 hours every night
and until noontime every weekend. The appropriate nursing
response is which of the following?
a.
b.
c.
d.

"The child should not be staying up so late at night."


"Adolescents need that amount of sleep every night."
"If the child eats properly, that should not be happening."
"The child probably is anemic and should eat more foods
containing iron."

65.
A clinic nurse provides information to the mother of a toddler
regarding toilet training. Which statement by the mother indicates
a need for further information regarding toilet training?
a. "Bladder control usually is achieved before bowel
control."
b. "The child should not be forced to sit on the potty for long
periods."
c. "The ability of the child to remove clothing is a sign of
physical readiness."
d. "The child will not be ready to toilet train until the age of
about 18 to 24 months."
66.
A nurse in the pediatric unit is admitting a 2-year-old child.
The nurse plans care, knowing that the child is in which stage of
Erikson's psychosocial stages of development?
a.
b.
c.
d.

Trust versus Mistrust


Autonomy versus Shame and Doubt
Initiative versus Guilt
Industry versus Inferiority

67.
During a home care visit, an older client complains of chronic
constipation. The nurse tells the client to:
a. Include rice and bananas in the diet.
b. Increase the intake of sugar-free products.
c. Increase fluid intake to at least eight glasses a day
and increase dietary fiber.
d. Increase potassium in the diet.

68.
A nurse at a well-baby clinic is providing nutrition instructions
to the mother of a 1-month-old infant. The nurse should instruct
the mother:
a.
b.
c.
d.

To offer rice cereal mixed with breast milk or formula


To introduce strained vegetables one at a time
To introduce strained fruits one at a time
That breast milk or formula is the main food

69.
The client is diagnosed with a disorder involving the inner ear.
Which of the following is the most common client complaint
associated with a disorder involving this part of the ear?
a.
b.
c.
d.

Pruritus
Tinnitus
Hearing loss
Burning in the ear

70.
The nurse notes that the physician has documented a
diagnosis of presbycusis on the client's chart. The nurse plans
care knowing that the condition is:
a.
b.
c.
d.

Tinnitus that occurs with aging


Nystagmus that occurs with aging
A conductive hearing loss that occurs with aging
A sensorineural hearing loss that occurs with aging

71.
A client arrives at the emergency department with a foreign
body in the left ear and tells the nurse that an insect flew into the
ear. Which intervention should the nurse implement initially?
a.
b.
c.
d.

Irrigation of the ear


Instillation of mineral oil
Instillation of antibiotic eardrops
Instillation of corticosteroid ointment

72.
A nurse is caring for a client with acute otitis media. In order
to reduce pressure and allow fluid to drain, the nurse anticipates
that which of the following would most likely be recommended to
the client?
a.
b.
c.
d.

The administration of diphenhydramine (Benadryl) capsules


A myringotomy
Strict bedrest
A mastoidectomy

73.
A nurse is assigned to care for a client after a mastoidectomy.
Which nursing intervention would be appropriate in the care of
this client?
a.
b.
c.
d.

Maintain a supine position.


Position the client on the affected side to promote drainage.
Change the ear dressing daily.
Monitor for signs of facial nerve injury.

74.
A nurse instructs a client in the use of a hearing aid. The nurse
includes which of the following instructions?
a. Check the battery to ensure that it is working before
use.
b. Leave the hearing aid in place while showering.
c. Hearing aids do not require any care.
d. A water-soluble lubricant is used on the hearing aid before
insertion.
75.
A nurse has given a client who is at risk for motion sickness
suggestions about medications that can prevent an occurrence.
The nurse determines that the client has correctly learned the
information if the client states that the medication is taken at
what time before the triggering event?
a.
b.
c.
d.

At least a half-day before


At least 1 hour before
At least the day before
At least 2 days before

76.
An adult client makes an appointment with an ear specialist
because of the frequent recurrence of middle ear infections. In
performing an intake assessment of the client, the nurse would
ask about which of the following as a risk factor related to
infection of the ears?
a.
b.
c.
d.

Exposure to loud noise


Use of drilling and other power tools
Congenital abnormalities
Occupational noise

77.
A nurse is educating a client on how to eliminate whistling
from a hearing aid. The nurse recognizes that additional
instruction is needed when the client states which of the
following?
a. "I will cleanse my ear mold."

b. "I will try reinserting the hearing aid."


c. "I will raise the volume of my hearing aid."
d. "I will make sure that my hair is not caught between the ear
mold and canal."
78.
The nurse is reviewing the physician's prescriptions for a client
with Meniere's disease. Which of the following diets would most
likely be prescribed for the client?
a.
b.
c.
d.

Low-fat diet
Low-sodium diet
Low-cholesterol diet
Low-carbohydrate diet

79.
The nurse is preparing to care for a client with acquired
immunodeficiency syndrome (AIDS) who has Pneumocystis
jiroveci pneumonia. In planning infection control for this client,
which of the following would be the appropriate form of isolation
to use to prevent the spread of infection to others?
a.
b.
c.
d.

Strict isolation
Contact precautions
Enteric precautions
Blood and body fluid precautions

80.
A client has a prescription for continuous monitoring of oxygen
saturation by pulse oximetry. The nurse performs which of the
following as the best action to ensure accurate readings on the
oximeter?
a. Apply the sensor to a finger that is cool to the touch.
b. Place the sensor distal to an intravenous (IV) site with a
continuous IV infusion.
c. Apply the sensor to a finger with very dark nail polish.
d. Ask the client to limit motion in the hand attached to
the pulse oximeter.
81.
A community health nurse is providing an educational session
to a group of community members at a local high school
regarding the issue of organ donation. A member of the group
asks the nurse, "How old does someone have to be to provide
consent for organ donation?"; The nurse accurately responds by
telling the member:
a. A person can sign papers to become a donor at 16 years of
age.
b. Written consent is never required to become a donor.

c. A donor must be 18 years of age or older to provide


consent.
d. The family is responsible for decision making about organ
donation at the time of death.
82.
A client who suffered a severe head injury has had vigorous
treatment to control cerebral edema. Brain death has been
determined. The nurse prepares to carry out which of the
following measures to maintain viability of the kidneys before
organ donation?
a.
b.
c.
d.

Monitoring temperature
Administering intravenous (IV) fluids
Assessing lung sounds
Performing range-of-motion exercises to the extremities

83.
The hospice nurse visits a client who is dying of ovarian
cancer. During the visit, the client says, "If I can just live long
enough to attend my daughter's graduation, I'll be ready to die."
Which of the following phases of coping is this client
experiencing?
a.
b.
c.
d.

Anger
Denial
Bargaining
Depression

84.
A nurse is caring for a client who is terminally ill. When
assessing the client, the nurse recognizes which of the following
as the most common distress symptom near the end of life?
a.
b.
c.
d.

Pain
Withdrawal
Anxiety
Depression

85.
The wife of a terminally ill client steps out of his room in tears.
She tells the nurse, "I don't know what I'm going to do when he's
gone!" What is the nurse's best response?
a.
b.
c.
d.

"I know. It will get easier with time."


"Don't worry, things will be fine."
"You need to be strong for him! Don't cry."
"This must be very hard for you."

86.
A client is brought to the emergency department in an
unresponsive state, and a diagnosis of hyperglycemic

hyperosmolar nonketotic syndrome is made. The nurse would


immediately prepare to initiate which of the following anticipated
physician's prescriptions?
a.
b.
c.
d.

Endotracheal intubation
100 units of NPH insulin
Intravenous infusion of normal saline
Intravenous infusion of sodium bicarbonate

87.
A nurse is monitoring a client newly diagnosed with diabetes
mellitus for signs of complications. Which of the following, if
exhibited in the client, would indicate hyperglycemia and warrant
physician notification?
a.
b.
c.
d.

Polyuria
Diaphoresis
Hypertension
Increased pulse rate

88.
A client is admitted to an emergency department, and a
diagnosis of myxedema coma is made. Which action would the
nurse prepare to carry out initially?
a.
b.
c.
d.

Warm the client.


Maintain a patent airway.
Administer thyroid hormone.
Administer fluid replacement.

89.
A nurse is caring for a postoperative parathyroidectomy client.
Which client complaint would indicate that a serious, lifethreatening complication may be developing, requiring immediate
notification of the physician?
a.
b.
c.
d.

Laryngeal stridor
Abdominal cramps
Difficulty in voiding
Mild to moderate incisional pain

90.
A nurse is performing an assessment on a client with
pheochromocytoma. Which of the following assessment data
would indicate a potential complication associated with this
disorder?
a.
b.
c.
d.

A coagulation time of 5 minutes


A urinary output of 50 mL per hour
A blood urea nitrogen level of 20 mg/dL
A heart rate that is 90 beats/min and irregular

91.
A client received 20 units of Humulin NPH insulin
subcutaneously at 8:00 AM. A nurse should assess the client for a
hypoglycemic reaction at:
a.
b.
c.
d.

10:00 AM
11:00 AM
5:00 PM
11:00 PM

92.
A client with type 1 diabetes mellitus is to begin an exercise
program, and the nurse is providing instructions regarding the
program. Which of the following does the nurse include in the
teaching plan?
a.
b.
c.
d.

Exercise is best performed during peak times of insulin.


Administer insulin after exercising.
Take a blood glucose test before exercising.
Try to exercise before mealtime.

93.
A nurse is caring for a client with a diagnosis of Addison's
disease. The nurse is monitoring the client for signs of Addisonian
crisis. The nurse should assess the client for which manifestation
that would be associated with this crisis?
a.
b.
c.
d.

Diaphoresis
Agitation
Restlessness
Severe abdominal pain

94.
A nurse is developing a plan of care for a client who is
scheduled for a thyroidectomy. The nurse develops a nursing
diagnosis related to psychosocial needs, knowing that which of
the following is likely to occur in the client?
a.
b.
c.
d.

Body image changes


Infertility
Sexual dysfunction
Gynecomastia

95.
A nurse is caring for a client after thyroidectomy. The client
expresses concern about the postoperative voice hoarseness she
is experiencing and asks if the hoarseness will subside. The nurse
explains that the hoarseness:
a. Indicates nerve damage
b. Is permanent
c. Will worsen before it subsides, which may take 6 months

d. Is normal during this time and will subside


96.
A client has abnormal amounts of circulating thyronine (T3)
and thyroxine (T4). The nurse plans care for the client anticipating
that the client may have a deficiency of which of the following
dietary elements?
a.
b.
c.
d.

Calcium
Magnesium
Phosphorus
Iodine

97.
A test to measure long-term control of diabetes mellitus has
been prescribed for a client. In instructing the client about the
test, the nurse explains that long-term control can be measured
because chronic high blood glucose levels lead to irreversible
glucose binding onto:
a.
b.
c.
d.

Muscle tissue
Adipose tissue
Red blood cells
Platelets

98.
A nurse hears a client calling out for help, hurries down the
hallway to the client's room, and finds the client lying on the floor.
The nurse performs a thorough assessment, assists the client
back to bed, notifies the physician of the incident, and completes
an incident report. Which of the following should the nurse
document on the incident report?
a.
b.
c.
d.

The client fell out of bed.


The client climbed over the side rails.
The client was found lying on the floor.
The client became restless and tried to get out of bed.

99.
A registered nurse arrives at work and is told to report (float)
to the intensive care unit (ICU) for the day because the ICU is
understaffed and needs additional nurses to care for the clients.
The nurse has never worked in the ICU. The nurse should take
which action first?
a.
b.
c.
d.

Call the hospital lawyer.


Refuse to float to the ICU.
Call the nursing supervisor.
Report to the ICU and identify tasks that can be
performed safely.

100. A nurse who works on the night shift enters the medication
room and finds a co-worker with a tourniquet wrapped around the
upper arm. The co-worker is about to insert a needle, attached to
a syringe containing a clear liquid, into the antecubital area. The
appropriate initial action by the nurse is which of the following?
a.
b.
c.
d.

Call security.
Call the police.
Call the nursing supervisor.
Lock the co-worker in the medication room until help is
obtained.

101. A nurse has made an error in a narrative documentation of an


assessment finding on a client and obtains the client's record to
correct the error. The nurse corrects the error by:
a. Documenting a late entry into the client's record
b. Trying to erase the error for space to write in the correct data
c. Using correction fluid to delete the error to write in the
correct data
d. Drawing one line through the error, initialing and
dating the line, and then documenting the correct
information
102. A nursing instructor delivers a lecture to nursing students
regarding the issue of client's rights and asks a nursing student to
identify a situation that represents an example of invasion of
client privacy. Which of the following, if identified by the student,
indicates an understanding of a violation of this client right?
a.
b.
c.
d.

Performing a procedure without consent


Threatening to give a client a medication
Telling the client that he or she cannot leave the hospital
Observing care provided to the client without the
client's permission

103. A client was admitted involuntarily to the mental health unit


because of episodes of extremely violent behavior. The client is
demanding to be discharged from the hospital, and a nurse does
not allow the client to leave. Which of the following represents the
legal ramifications associated with the nurse's behavior?
a.
b.
c.
d.

The nurse will be charged with assault.


The nurse will be charged with slander.
The nurse will be charged with imprisonment.
No charge will be made against the nurse because the
nurse's actions are reasonable.

104. A nurse manager is planning to implement a change in the


nursing unit from team nursing to primary nursing. The nurse
anticipates that there will be resistance to the change during the
change process. The primary technique that the nurse would use
in implementing this change is which of the following?
a.
b.
c.
d.

Introduce the change gradually.


Confront the individuals involved in the change process.
Use coercion to implement the change.
Manipulate the participants in the change process.

105. During a home visit, the nurse discusses health care concerns
with the client and establishes mutual goals to help the client
become more independent. In this role, the nurse is functioning
as:
a.
b.
c.
d.

A researcher
A resource linker
An advocate
A collaborator

106. A day care nurse is observing a 2-year-old child and suspects


that the child may have strabismus. Which observation made by
the nurse might indicate this condition?
a.
b.
c.
d.

The child has difficulty hearing.


The child consistently tilts the head to see.
The child consistently turns the head to see.
The child does not respond when spoken to.

107. A nurse is reviewing the laboratory results for a child


scheduled for tonsillectomy. The nurse determines that which
laboratory value is most significant to review?
a.
b.
c.
d.

Creatinine level
Prothrombin time
Sedimentation rate
Blood urea nitrogen level

108. A nurse is caring for a child after a tonsillectomy. The nurse


monitors the child, knowing that which of the following indicates
that the child is bleeding?
a.
b.
c.
d.

Frequent swallowing
A decreased pulse rate
Complaints of discomfort
An elevation in blood pressure

109. A new mother expresses concern to a nurse regarding sudden


infant death syndrome (SIDS). She asks the nurse how to position
her new infant for sleep. The nurse appropriately tells the mother
that the infant should be placed on the:
a.
b.
c.
d.

Side or prone
Back or prone
Stomach with the face turned
Back rather than on the stomach

110. A physician has prescribed oxygen as needed for an infant


with congestive heart failure. In which situation should the nurse
administer the oxygen to the infant?
a.
b.
c.
d.

During sleep
When changing the infant's diapers
When the mother is holding the infant
When drawing blood for electrolyte level testing

111. A child with croup is being discharged from the hospital. The
nurse provides instructions to the mother and advises the mother
to bring the child to the emergency department if which of the
following occurs?
a.
b.
c.
d.

The child appears tired.


The child takes fluids poorly.
The child is irritable.
The child develops stridor.

112. The nurse is developing a teaching plan for the client with
glaucoma. Which of the following instructions would the nurse
include in the plan of care?
a. Avoid overuse of the eyes.
b. Decrease the amount of salt in the diet.
c. Eye medications will need to be administered for the
client's entire life.
d. Decrease fluid intake to control the intraocular pressure.
113. The nurse is performing an assessment on a client with a
suspected diagnosis of cataract. The chief clinical manifestation
that the nurse would expect to note in the early stages of cataract
formation is:
a. Diplopia
b. Eye pain
c. Floating spots

d. Blurred vision
114. The client sustains a contusion of the eyeball following a
traumatic injury with a blunt object. Which intervention is initiated
immediately?
a.
b.
c.
d.

Notify the physician.


Apply ice to the affected eye.
Irrigate the eye with cool water.
Accompany the client to the emergency department.

115. A client is diagnosed with glaucoma. Which of the following


assessment data gathered by a nurse identifies a risk factor
associated with this eye disorder?
a.
b.
c.
d.

A history of migraine headaches


Frequent urinary tract infections
Cardiovascular disease
Frequent upper respiratory infections

116. A male client arrives at the hospital emergency department


and tells the nurse that there is something in his eye. The nurse
looks into the client's eye and notes that the foreign body is
visible and is not embedded. The appropriate nursing action is
which of the following?
a.
b.
c.
d.

Irrigate the eye with normal saline.


Tell the client that the surgeon will need to be called.
Tell the client that the object will work its way out.
Touch the object gently with a cotton swab and lift it
out.

117. Immediately after cataract repair, a nurse notes that the


conjunctiva and eyelids on the client's operated eye are
edematous. The nurse interprets this finding as:
a.
b.
c.
d.

Grossly abnormal and should be reported at once


Normal and should subside within 3 days
Abnormal because the conjunctiva should not be affected
Abnormal because only the eyelids should be affected

118. A nurse is preparing to care for a client with a potassium


deficit. The nurse reviews the client's record and determines that
the client was at risk for developing the potassium deficit because
the client:
a. Sustained tissue damage
b. Requires nasogastric suction
c. Has a history of Addison's disease

d. Is taking a potassium-sparing diuretic


119. A nurse caring for a group of clients reviews the electrolyte
laboratory results and notes a potassium level of 5.5 mEq/L on
one client's laboratory report. The nurse understands that which
client is at highest risk for the development of a potassium value
at this level?
a.
b.
c.
d.

The client with colitis


The client with Cushing's syndrome
The client who has been overusing laxatives
The client who has sustained a traumatic burn

120. A nurse is assigned to care for a group of clients. On review of


the clients' medical records, the nurse determines that which
client is at risk for a fluid volume deficit?
a.
b.
c.
d.

A client with a colostomy


A client with congestive heart failure
A client on long-term corticosteroid therapy
A client receiving frequent wound irrigations

121. A nurse is caring for a client with a nasogastric tube that is


attached to low suction. The nurse monitors the client, knowing
that the client is at risk for which acid-base disorder?
a.
b.
c.
d.

Metabolic acidosis
Metabolic alkalosis
Respiratory acidosis
Respiratory alkalosis

122. A client with a history of lung disease is at risk for developing


respiratory acidosis. A nurse assesses this client for which signs
and symptoms characteristic of this disorder?
a.
b.
c.
d.

Bradycardia and hyperactivity


Decreased respiratory rate and depth
Headache, restlessness, and confusion
Bradypnea, dizziness, and paresthesias

123. A client is receiving an intravenous infusion of 1000 mL of


normal saline with 40 mEq of potassium chloride. The care unit
nurse is monitoring the client for signs of hyperkalemia. Which of
the following initially will be noted in the client if hyperkalemia is
present?
a. Mental status changes

b. Confusion
c. Muscle weakness
d. Depressed deep tendon reflexes
124. The nurse is assessing a client who is experiencing an acute
episode of cholecystitis. Where should the nurse anticipate the
location of the pain?
a. Right lower quadrant, radiating to the back
b. Right lower quadrant, radiating to the umbilicus
c. Right upper quadrant, radiating to the left scapula and
shoulder
d. Right upper quadrant, radiating to the right scapula
and shoulder
125. A client has just had a hemorrhoidectomy. What nursing
intervention is appropriate for this client?
a. Instruct the client to limit fluid intake to avoid urinary
retention.
b. Instruct the client to eat low-fiber foods to decrease the bulk
of the stool.
c. Apply and maintain ice packs over the dressing until
the packing is removed.
d. Help the client to a Fowler's position to place pressure on the
rectal area and decrease bleeding.
126. The physician has determined that the client with hepatitis
has contracted the infection from contaminated food. The nurse
understands that this client is most likely experiencing what type
of hepatitis?
a.
b.
c.
d.

Hepatitis A
Hepatitis B
Hepatitis C
Hepatitis D

127. The nurse is monitoring a client for the early signs and
symptoms of dumping syndrome. Which of the following indicate
this occurrence?
a.
b.
c.
d.

Sweating and pallor


Bradycardia and indigestion
Double vision and chest pain
Abdominal cramping and pain

128. A nurse is caring for a client with acute pancreatitis and is


monitoring the client for paralytic ileus. Which assessment data
would alert the nurse to this occurrence?
a. Firm, nontender mass palpable at the lower right costal
margin
b. Severe, constant pain with rapid onset
c. Inability to pass flatus
d. Loss of anal sphincter control
129. A client is scheduled for an oral cholecystogram. The nurse
should plan to prescribe which type of diet for the evening meal
before the test?
a.
b.
c.
d.

Low protein
High carbohydrate
Low fat
Liquid

130. The nurse notes documentation that a client is exhibiting


Cheyne-Stokes respirations. On assessment of the client, the
nurse expects to note which of the following?
a.
b.
c.
d.

Rhythmic respirations with periods of apnea


Regular rapid and deep, sustained respirations
Totally irregular respiration in rhythm and depth
Irregular respirations with pauses at the end of inspiration
and expiration

131. The nurse is instructing a client how to perform a testicular


self-examination (TSE). The nurse explains that the best time to
perform this exam is:
a.
b.
c.
d.

After a shower or bath


While standing to void
After having a bowel movement
While lying in bed before arising

132. The community health nurse is instructing a group of female


clients about breast self-examination. The nurse instructs the
clients to perform the examination:
a.
b.
c.
d.

At the onset of menstruation


Every month during ovulation
Weekly at the same time of day
1 week after menstruation begins

133. A nurse is testing a client for astereognosis. The nurse should


ask the client to close the eyes and do which of the following?
a. Identify three numbers or letters traced in the client's palm.
b. Identify three objects placed in the hand one at a
time.
c. State whether one or two pinpricks are felt when the skin is
pricked bilaterally in the same place.
d. Identify the smallest distance between two detectable
pinpricks, made with two pins held at various distances.
134. A nurse is preparing to perform a Weber test on a client. The
nurse obtains which item needed to perform this test?
a.
b.
c.
d.

A tongue blade
A stethoscope
A tuning fork
A reflex hammer

135. A chest x-ray report for a client documents the presence of a


left apical pneumothorax. The nurse would assess the status of
breath sounds in that area by placing the stethoscope:
a.
b.
c.
d.

Posteriorly under the left scapula


Just under the left clavicle
Over the fifth intercostal space
Near the lateral twelfth rib

136. A nurse is providing home care instructions to the mother of a


10-year-old child with hemophilia. Which of the following activities
should the nurse suggest that the child could participate in safely
with peers?
a.
b.
c.
d.

Soccer
Basketball
Swimming
Field hockey

137. Laboratory studies are performed for a child suspected to


have iron deficiency anemia. The nurse reviews the laboratory
results, knowing that which of the following results would indicate
this type of anemia?
a.
b.
c.
d.

Elevated hemoglobin level


Decreased reticulocyte count
Elevated red blood cell count
Red blood cells that are microcytic and hypochromic

138. The nurse analyzes the laboratory values of a child with


leukemia who is receiving chemotherapy. The nurse notes that
the platelet count is 19,500 cells/mm3. Based on this laboratory
result, which intervention would the nurse document in the plan
of care?
a.
b.
c.
d.

Monitor closely for signs of infection.


Monitor the temperature every 4 hours.
Initiate protective isolation precautions.
Use a soft small toothbrush for mouth care.

139. A child is brought to the emergency department after being


accidentally struck in the lower back region with a baseball bat.
When gathering assessment data, the nurse discovers that the
child has hemophilia. The nurse should immediately assess:
a.
b.
c.
d.

The urine for hematuria


The respiratory rate
For complaints of headache
For signs of slurred speech

140. A pediatric nurse educator provides a teaching session to the


nursing staff regarding hemophilia. Which of the following
statements regarding this disorder would the nurse plan to
include in the discussion?
a.
b.
c.
d.

Males inherit hemophilia from their fathers.


Hemophilia is a Y-linked hereditary disorder.
Females inherit hemophilia from their mothers.
Hemophilia A results from deficiency of factor VIII.

141. A client is suspected of having systemic lupus erythematous.


The nurse monitors the client, knowing that which of the following
is one of the initial characteristic signs of systemic lupus
erythematous?
a.
b.
c.
d.

Weight gain
Subnormal temperature
Elevated red blood cell count
Rash on the face across the bridge of the nose and on
the cheeks

142. The nurse is assisting in planning care for a client with a


diagnosis of immunodeficiency. The nurse would incorporate
which of the following as a priority in the plan of care?
a. Protecting the client from infection
b. Providing emotional support to decrease fear

c. Encouraging discussion about lifestyle changes


d. Identifying factors that decreased the immune function
143. The client with acquired immunodeficiency syndrome is
diagnosed with cutaneous Kaposi's sarcoma. Based on this
diagnosis, the nurse understands that this has been confirmed by
which of the following?
a.
b.
c.
d.

Swelling in the genital area


Swelling in the lower extremities
Punch biopsy of the cutaneous lesions
Appearance of reddish-blue lesions noted on the skin

144. The client with acquired immunodeficiency syndrome has


begun therapy with zidovudine (Retrovir, azidothymidine, AZT,
ZDV). The nurse carefully monitors which of the following
laboratory results during treatment with this medication?
a.
b.
c.
d.

Blood culture
Blood glucose level
Complete blood count
Blood urea nitrogen level

145. A nursing student is conducting a clinical conference on


immunity. In discussing active versus passive immunity, the
student emphasizes that active immunity:
a.
b.
c.
d.

Lasts much longer then passive immunity


Is less effective at preventing subsequent infections
Provides protection immediately
Has a half-life of about 30 days

146. A nursing student is describing the differences between


specific and nonspecific immunity. The student correctly identifies
specific immunity by stating that this type of immunity is:
a.
b.
c.
d.

The first line of defense against infection


The type of immunity that reacts the same to all antigens
The second line of defense against infection
Present and functioning at birth

147. The mother of a 6-year-old child arrives at a clinic because the


child has been experiencing scratchy, red, and swollen eyes. The
nurse notes a discharge from the eyes and sends a culture to the
laboratory for analysis. Chlamydial conjunctivitis is diagnosed.
Based on this diagnosis, the nurse determines that which of the
following requires further investigation?

a.
b.
c.
d.

Possible trauma
Possible sexual abuse
Presence of an allergy
Presence of a respiratory infection

148. A mother of a 3-year-old child arrives at a clinic and tells a


nurse that the child has been scratching the skin continuously
and has developed a rash. The nurse assesses the child and
suspects the presence of scabies. The nurse bases this suspicion
on which finding noted on assessment of the child's skin?
a.
b.
c.
d.

Fine grayish red lines


Purple-colored lesions
Thick, honey-colored crusts
Clusters of fluid-filled vesicles

149. A clinic nurse is reviewing the physician's prescription for a


child who has been diagnosed with scabies. Lindane has been
prescribed for the child. The nurse questions the prescription if
which of the following is noted in the child's record?
a.
b.
c.
d.

The child is 18 months old.


The child is being bottle-fed.
A sibling is using lindane for the treatment of scabies.
The child has a history of frequent respiratory infections.

150. A clinic nurse prepares to administer a measles, mumps,


rubella (MMR) vaccine to a 5-year-old child. The nurse administers
this vaccine by which best route and in which best site?
a.
b.
c.
d.

Intramuscularly in the deltoid muscle


Subcutaneously in the gluteal muscle
Subcutaneously in the outer aspect of the upper arm
Intramuscularly in the anterolateral aspect of the thigh

151. A nursing student is assigned to administer immunizations to


children in a clinic. The nursing instructor asks the student about
related precautions and general contraindications to
immunization. The student responds correctly by telling the
instructor that the presence of which condition would be a reason
to delay an immunization in a child?
a.
b.
c.
d.

A cold
Otitis media
Mild diarrhea
A severe febrile illness

152. The client is being admitted to the hospital for treatment of


acute cellulitis of the lower left leg. The client asks the admitting
nurse to explain what cellulitis means. The nurse bases the
response on the understanding that the characteristics of cellulitis
include:
a. An inflammation of the epidermis only
b. A skin infection into the dermis and subcutaneous
tissue
c. An acute superficial infection of the dermis and lymphatics
d. An epidermal and lymphatic infection caused by
Staphylococcus
153. The clinic nurse notes that the physician has documented a
diagnosis of herpes zoster (shingles) in the client's chart. Based
on an understanding of the cause of this disorder, the nurse
determines that this definitive diagnosis was made following
which diagnostic test?
a.
b.
c.
d.

Patch test
Skin biopsy
Culture of the lesion
Wood's light examination

154. The evening nurse reviews the nursing documentation in the


client's chart and notes that the day nurse has documented that
the client has a stage II pressure ulcer in the sacral area. Which of
the following would the nurse expect to note on assessment of
the client's sacral area?
a.
b.
c.
d.

Intact skin
Full-thickness skin loss
Exposed bone, tendon, or muscle
Partial-thickness skin loss of the dermis

155. A nursing student prepares a plan of care for a client with a


diagnosis of acute cellulitis of the lower leg. On reviewing the plan
of care, the nursing instructor determines that the student
understands the treatment measures if the care plan includes:
a.
b.
c.
d.

Warm compresses to the affected area


Cold compresses to the affected area
Heat lamp treatments four times daily
Alternating hot to cold compresses every 2 hours

156. A client who is being evaluated for thermal burn injuries to the
arms and legs complains of thirst and asks the nurse for a drink.
Which action by the nurse is appropriate?

a.
b.
c.
d.

Give the client small glasses of clear liquids.


Keep the client on NPO status.
Allow the client to have full liquids.
Order the client a full meal tray with extra liquids.

157. A nurse is assessing a client who has undergone


chemotherapy for cancer and notes that the client exhibits hair
loss. The nurse documents this finding as:
a.
b.
c.
d.

Hirsutism
Alopecia
Xerosis
Hyperhidrosis

158. A client is diagnosed with psoriasis. The nurse plans care,


knowing that this disorder is the result of abnormalities of which
of the following processes?
a.
b.
c.
d.

Increased vascularity
Excess keratin formation
Decreased vitamin D synthesis
Defects in the sebaceous glands

159. A nurse is caring for a client in labor. The nurse determines


that the client is beginning the second stage of labor when which
of the following assessments is noted?
a.
b.
c.
d.

The contractions are regular.


The membranes have ruptured.
The cervix is dilated completely.
The client begins to expel clear vaginal fluid.

160. After an amniotomy has been performed, a nurse should first


assess:
a.
b.
c.
d.

For cervical dilation


For bladder distention
The maternal blood pressure
The fetal heart rate pattern

161. An ultrasound is performed on a client at term gestation who


is experiencing moderate vaginal bleeding. The results of the
ultrasound indicate that abruptio placentae is present. Based on
these findings, the nurse would prepare the client for:
a. Delivery of the fetus
b. Strict monitoring of intake and output
c. Complete bed rest for the remainder of the pregnancy

d. The need for weekly monitoring of coagulation studies until


the time of delivery
162. A nurse has developed a plan of care for a client experiencing
dystocia and includes several nursing interventions in the plan of
care. The nurse prioritizes the plan of care and selects which
intervention as the highest priority?
a.
b.
c.
d.

Providing comfort measures


Monitoring the fetal heart rate
Changing the client's position frequently
Keeping the significant other informed of the progress of the
labor

163. A nurse in a labor room is assisting with the vaginal delivery


of a newborn infant. The nurse would monitor the client closely for
the risk of uterine rupture if which of the following occurred?
a. Forceps delivery
b. Schultz presentation
c. Hypotonic contractions
d. Weak bearing-down efforts
164. A client in labor is dilated 10 cm. At this point of the labor
process, the nurse would plan to assess and document the fetal
heart rate at least:
a.
b.
c.
d.

Hourly
Every 15 minutes
Every 30 minutes
Before each contraction

165. During the intrapartum period, a nurse is caring for a client


with sickle cell disease. The nurse ensures that the client receives
adequate intravenous fluid intake and oxygen consumption
primarily to:
a. Stimulate the labor process.
b. Prevent dehydration and hypoxemia.
c. Avoid the necessity of a cesarean delivery.
d. Eliminate the need for analgesic administration.
166. A client arrives in the emergency room complaining of chest
pain that began 4 hours ago. A troponin T blood specimen is
obtained, and the results indicate a level of 0.6 ng/mL. The nurse
determines that this result indicates a:
a. Normal level
b. Low value that indicates possible gastritis
c. Level that indicates a myocardial infarction

d. Level that indicates the presence of possible angina


167. An adult client with cirrhosis has been following a diet with
optimal amounts of protein because neither an excess nor a
deficiency of protein has been helpful. The nurse evaluates the
client's status as being most satisfactory if the total protein level
is which of the following values?
a. 0.4 g/dL
b. 3.7 g/dL
c. 6.4 g/dL
d. 9.8 g/dL
168. An adult female client has a hemoglobin level of 10.8 g/dL.
The nurse interprets that this result is most likely caused by which
of the following conditions noted in the client's history?
a.
b.
c.
d.

Dehydration
Heart failure
Iron deficiency anemia
Chronic obstructive pulmonary disease

169. A client is undergoing diagnostic testing because of suspected


renal disease. Which of the following laboratory tests best
evaluates the kidneys' ability to regulate fluid balance?
a. Urine specific gravity
b. Blood urea nitrogen (BUN)
c. Creatinine
d. Urinary protein
170. A clinic nurse has obtained a throat culture specimen from a
client in whom a throat infection is suspected. The nurse calls the
laboratory to have the specimen picked up and is told that the
laboratory is short-staffed and that the laboratory assistant will
pick up the specimen in 2 hours. Which of the following is the
appropriate nursing action?
a. Tell the client to return in 1 hour for a repeat throat culture.
b. Refrigerate the specimen.
c. Tell the laboratory that someone needs to pick up the
specimen immediately.
d. Contact the physician who ordered the specimen.
171. To detect the development of a chronic carrier state in a client
with hepatitis, the nurse should assess the client's serum
laboratory results for:
a. Antibody to surface antigen (anti-HBs)
b. Hepatitis B surface antigen (HBsAg)
c. Hepatitis B virus DNA

d. Prolonged prothrombin time


172. Which of the following outcomes would the nurse expect to
observe in the client who is recovering from viral hepatitis without
complications?
a.
b.
c.
d.

Decreased absorption of vitamin K in the intestine


Increasing prothrombin time values
Presence of asterixis
Decrease in aspartate aminotransferase (AST)

173. A nurse is reviewing the arterial blood gas analysis results for
a client in the respiratory care unit and notes a pH of 7.38, PaCO2
of 38 mm Hg, PaO2 of 86 mm Hg, and HCO3- of 23 mEq/L. The
nurse interprets that these values indicate which of the following?
a.
b.
c.
d.

Normal results
Metabolic alkalosis
Metabolic acidosis
Respiratory acidosis

174. A nurse checks the laboratory results of a serum drug level


assay for a client taking digoxin (Lanoxin) 0.125 mg daily. Which
of the following values would indicate a therapeutic level?
a.
b.
c.
d.

0.1 ng/mL
0.3 ng/mL
1.8 ng/mL
2.4 ng/mL

175. A clinic nurse is reviewing the laboratory test results for a


client seen in the health care clinic. The nurse determines that the
white blood cell (WB count is normal if which of the following
values is noted on the laboratory report?
a.
b.
c.
d.

2000 cells/mm3
3800 cells/mm3
8400 cells/mm3
12,500 cells/mm3

176. The nurse is making initial rounds on the nursing unit to


assess the condition of assigned clients. The nurse notes that a
client's intravenous (IV) site is cool, pale, and swollen, and the
solution is not infusing. The nurse concludes that which of the
following complications has occurred?
a. Infection
b. Phlebitis

c. Infiltration
d. Thrombosis
177. The nurse is inserting an intravenous line into a client's vein.
After the initial stick, the nurse continues to advance the catheter
if:
a. The catheter advances easily.
b. The vein is distended under the needle.
c. The client does not complain of discomfort.
d. Blood return shows in the backflash chamber of the
catheter.
178. The nurse notes that the site of a client's peripheral
intravenous (IV) catheter is reddened, warm, painful, and slightly
edematous proximal to the insertion point of the IV catheter. After
taking appropriate steps to care for the client, the nurse
documents in the medical record that the client experienced:
a.
b.
c.
d.

Phlebitis of the vein


Infiltration of the IV line
Hypersensitivity to the IV solution
Allergic reaction to the IV catheter material

179. The nurse listening to morning report learns that an assigned


client received a unit of granulocytes the previous evening. The
nurse makes a note to assess the results of which of the following
daily serum laboratory studies to assess the effectiveness of the
transfusion?
a.
b.
c.
d.

Hematocrit level
Erythrocyte count
Hemoglobin level
White blood cell count

180. A nursing student is assigned to administer an iron injection to


a client. The co-assigned nurse asks the student about the
technique for administration of this medication. The student
indicates understanding of the administration procedure by
identifying the correct injection site and method as:
a.
b.
c.
d.

Anterolateral thigh using an air lock


Gluteal muscle using Z-track technique
Subcutaneous tissue of the abdomen using a 1-inch needle
Deltoid muscle using a 1-inch needle

181. A nurse has an order to transfuse a unit of packed red blood


cells to a client who does not currently have an intravenous (IV)

line inserted. When obtaining supplies to start the IV infusion, the


nurse selects an angiocatheter with a size of:
a.
b.
c.
d.

18 gauge
21 gauge
22 gauge
24 gauge

182. A nurse is administering an acetaminophen (Tylenol)


suppository to a child with a fever. The nurse inserts the
suppository into the rectum a distance of no more than:
a.
b.
c.
d.

0.5 cm
1 cm
2 cm
2.5 cm

183. A nurse plans to administer a medication by intravenous (IV)


bolus through the primary IV line. The nurse notes that the
medication is incompatible with the primary IV solution. Which of
the following is the appropriate nursing action to safely administer
the medication?
a. Call the physician for a prescription to change the route of
the medication.
b. Start a new IV line for the medication.
c. Flush the tubing before and after the medication with
normal saline.
d. Flush the tubing before and after the medication with sterile
water.
184. The nurse is preparing to insert an intravenous (IV)
angiocatheter into a client's inner forearm. Before cannulating the
vein, the nurse cleanses the entry site by using which of the
following motions?
a.
b.
c.
d.

Scrubbing from the wrist toward the elbow


Scrubbing from the elbow toward the wrist
Using a circular motion from the center outward
Using a circular motion inward toward the center

185. Diphenhydramine hydrochloride (Benadryl), 25 mg orally


every 6 hours, is prescribed for a child with an allergic reaction.
The child weighs 25 kg. The safe pediatric dosage is 5 mg/kg per
day. The nurse determines which of the following concerning the
dose prescribed?
a. The dose prescribed is safe.

b. The dose prescribed is too low.


c. The dose prescribed is too high.
d. There is not enough information to determine the safe dose.
186. The home care nurse is visiting a client whose husband died 6
months ago. Which behavior by the client indicates ineffective
coping?
a.
b.
c.
d.

Neglecting her personal grooming


Looking at old snapshots of her family
Participating in a senior citizens' program
Visiting her husband's grave once a month

187. Unresolved feelings related to loss most likely may be


recognized during which phase of the therapeutic nurse-client
relationship?
a.
b.
c.
d.

Working
Trusting
Orientation
Termination

188. An 18-year-old woman is admitted to an inpatient mental


health unit with the diagnosis of anorexia nervosa. A cognitive
behavioral approach is used as part of her treatment plan. A
nurse understands that the purpose of this approach is to:
a.
b.
c.
d.

Provide a supportive environment.


Examine intrapsychic conflicts and past issues.
Emphasize social interaction with clients who withdraw.
Help the client identify and examine dysfunctional
thoughts and beliefs.

189. A nurse is caring for a client with a phobia who is being


treated for the condition. The client is introduced to short periods
of exposure to the phobic object while in a relaxed state. The
nurse understands that this form of behavior modification can
best be described as:
a.
b.
c.
d.

Milieu therapy
Aversion therapy
Self-control therapy
Systematic desensitization

190. A client is admitted to the mental health unit with a diagnosis


of schizophrenia. A nursing diagnosis formulated for the client is
disturbed thought processes related to paranoia. In formulating

nursing interventions with the members of the health care team,


a nurse provides instructions to:
a. Increase socialization of the client with peers.
b. Avoid laughing or whispering in front of the client.
c. Begin to educate the client about social supports in the
community.
d. Have the client sign a release of information to appropriate
parties so that adequate data can be obtained for
assessment purposes.
191. A nurse is caring for a client with a diagnosis of agoraphobia.
When communicating with the client about the disorder, the nurse
would expect the client to describe which of the following
behaviors?
a.
b.
c.
d.

A need to wash the hands several times before eating a meal


A fear of leaving the house
A fear of speaking in public
A fear of riding in elevators

192. The mother of a 6-year-old child who has type 1 diabetes


mellitus calls a clinic nurse and tells the nurse that the child has
been sick. The mother reports that she checked the child's urine
and it was positive for ketones. The nurse instructs the mother to:
a.
b.
c.
d.

Hold the next dose of insulin.


Come to the clinic immediately.
Administer an additional dose of regular insulin.
Encourage the child to drink calorie-free liquids.

193. A child has fluid volume deficit. The nurse performs an


assessment and determines that the child is improving and the
deficit is resolving if:
a. The child has no tears.
b. Urine specific gravity is 1.030.
c. Urine output is less than 1 mL/kg/hr.
d. Capillary refill is less than 2 seconds.
194. A 6-year-old child with diabetes mellitus and the child's
mother come to the health care clinic for a routine examination.
The nurse evaluates the data collected during this visit to
determine if the child has been euglycemic since the last visit.
Which information is the most significant indicator of euglycemia?
a. Daily glucose monitor log
b. Fasting blood glucose performed on the day of the clinic visit
c. Glycosylated hemoglobin

d. Dietary history for the previous week


195. The home care nurse is visiting a child newly diagnosed with
diabetes mellitus. The nurse is instructing the child and parents
regarding actions to take if hypoglycemic reactions occur. The
nurse tells the child to:
a. Drink 8 ounces of diet cola at the first sign of weakness.
b. Report to a hospital emergency department if the blood
glucose is 60 mg/dL.
c. Carry hard candies whenever leaving home in case a
hypoglycemic reaction occurs.
d. Administer glucagon immediately if shakiness is felt.
196. An adolescent with diabetes receives 30 units of isophane
insulin (neutral protamine Hagedorn, or NPH) at 7:00 AM. In
accordance with the peak insulin action time, the nurse would
monitor for a hypoglycemic episode:
a.
b.
c.
d.

At bedtime
At midmorning
Before supper
After breakfast

197. A clinic nurse is assessing a child for dehydration. The nurse


determines that the child is moderately dehydrated if which of the
following is noted on assessment?
a.
b.
c.
d.

Oliguria
Flat fontanels
Pale skin color
Moist mucous membranes

198. A nurse is conducting health screening for osteoporosis. Which


of the following clients is at greatest risk of developing this
disorder?
a.
b.
c.
d.

A 25-year-old woman who jogs


A 36-year-old man who has asthma
A 70-year-old man who consumes excess alcohol
A sedentary 65-year-old woman who smokes
cigarettes

199. A nurse is evaluating the pin sites of a client in skeletal


traction. The nurse would be least concerned with which of the
following findings?
a. Inflammation

b. Serous drainage
c. Pain at a pin site
d. Purulent drainage
200. A nurse is admitting a client with multiple trauma to the
nursing unit. The client has a leg fracture and had a plaster cast
applied. In positioning the casted leg, the nurse should:
a.
b.
c.
d.

Keep the leg in a level position.


Elevate the leg for 3 hours and put it flat for 1 hour.
Keep the leg level for 3 hours and elevate it for 1 hour.
Elevate the leg on pillows continuously for 24 to 48
hours.

201. A client with diabetes mellitus has had a right below-knee


amputation. The nurse would assess specifically for which of the
following signs because of the history of diabetes?
a. Hemorrhage
b. Edema of the stump
c. Slight redness of the incision
d. Separation of the wound edges
202. A nurse is preparing a plan of care for a client in skin traction.
The nurse includes in the plan that a priority intervention is to
assess the client frequently for:
a.
b.
c.
d.

The presence of bowel sounds


Signs of infection around the pin sites
Signs of skin breakdown
Urinary incontinence

203. A community health nurse is providing an educational session


for community members regarding dietary measures that will
assist in reducing the risk of osteoporosis. The nurse instructs the
community members to increase dietary intake of which food
known to be helpful in minimizing this risk?
a.
b.
c.
d.

Yogurt
Turkey
Spaghetti
Shellfish

204. A nurse is preparing to perform an assessment on a client


being admitted to the hospital with a diagnosis of sickle cell crisis.
Which of the following would the nurse expect to note on
assessment of the client?
a. Bradycardia

b. Blurred vision
c. Diarrhea
d. Joint swelling
205. A client with a 4-day-old lumbar vertebral fracture is
experiencing muscle spasms. The nurse avoids using which of the
following in an effort to relieve the spasm?
a.
b.
c.
d.

Prescribed intermittent traction


Analgesics
Heat
Ice

206. A nurse is caring for a child recently diagnosed with cerebral


palsy, and the parents of the child ask the nurse about the
disorder. The nurse bases her response on the understanding that
cerebral palsy is:
a. An infectious disease of the central nervous system
b. An inflammation of the brain as a result of a viral illness
c. A congenital condition that results in moderate to severe
retardation
d. A chronic disability characterized by impaired muscle
movement and posture
207. A nurse develops a plan of care for a child at risk for tonicclonic seizures. In the plan of care, the nurse identifies seizure
precautions and documents that which item(s) need to be placed
at the child's bedside?
a.
b.
c.
d.

Emergency cart
Tracheotomy set
Padded tongue blade
Suctioning equipment and oxygen

208. An emergency department nurse is performing an assessment


on a child suspected of being sexually abused. Which assessment
data obtained by the nurse most likely support this suspicion?
a.
b.
c.
d.

Poor hygiene
Difficulty walking
Fear of the parents
Bald spots on the scalp

209. A child with autism is being admitted to the hospital for


diagnostic tests. The nurse should assign this child to a:
a. Private room

b. Semiprivate room
c. Four-bed ward room
d. Contact isolation room
210. A child is brought to the hospital emergency department for
an injury to the lower right arm that occurred in a fall off a bicycle.
On assessment the nurse notes that the skin at the site of the
injury is intact. A fracture is suspected, and a radiograph is taken.
The nurse can see on the radiograph viewer that the fracture of
the bone is across the entire bone shaft with some possible
displacement. The nurse determines that this child's fracture is a:
a.
b.
c.
d.

Simple fracture
Compound fracture
Greenstick fracture
Comminuted fracture

211. A nurse is caring for a child who sustained a head injury after
falling from a tree. On assessment of the child, the nurse notes
the presence of a watery discharge from the child's nose. The
nurse will immediately test the discharge for the presence of
which of the following substance?
a.
b.
c.
d.

Glucose
Protein
White blood cells
Neutrophils

212. A child is admitted to the hospital with a diagnosis of acute


bacterial meningitis. In reviewing the physician's prescriptions,
which of the following would the nurse question as appropriate for
a child with this diagnosis?
a.
b.
c.
d.

Maintain strict intake and output.


Draw blood for a culture and sensitivity.
Administer an oral antibiotic.
Place the child on droplet precautions in a private room.

213. The nurse is assessing the motor function of an unconscious


client. The nurse would plan to use which of the following to test
the client's peripheral response to pain?
a.
b.
c.
d.

Sternal rub
Nail bed pressure
Pressure on the orbital rim
Squeezing of the sternocleidomastoid muscle

214. The nurse is evaluating the neurological signs of a client in


spinal shock following spinal cord injury. Which of the following
observations by the nurse indicates that spinal shock persists?
a. Hyperreflexia
b. Positive reflexes
c. Reflex emptying of the bladder
d. Flaccid paralysis
215. The nurse is teaching the client with myasthenia gravis about
the prevention of myasthenic and cholinergic crises. The nurse
tells the client that this is most effectively done by:
a.
b.
c.
d.

Eating large, well-balanced meals


Doing muscle-strengthening exercises
Doing all chores early in the day while less fatigued
Taking medications on time to maintain therapeutic
blood levels

216. A client has dysfunction of the cochlear division of the


vestibulocochlear nerve (cranial nerve VIII). The nurse would
determine that the client is adequately adapting to this problem if
the client states a plan to obtain:
a.
b.
c.
d.

A hearing aid
A walker
Eyeglasses
A bath thermometer

217. The nurse is conducting a dietary assessment on a client who


is on a vegan diet. The nurse provides dietary teaching focusing
on foods high in which vitamin that may be lacking in a vegan
diet?
a.
b.
c.
d.

Vitamin A
Vitamin B12
Vitamin C
Vitamin E

218. A client is recovering from abdominal surgery and has a large


abdominal wound. A nurse encourages the client to eat which
food item that is naturally high in vitamin C to promote wound
healing?
a.
b.
c.
d.

Milk
Oranges
Bananas
Chicken

219. A client has been discharged to home on parenteral nutrition


(PN). With each visit, a home care nurse assesses which of the
following parameters most closely in monitoring this therapy?
a.
b.
c.
d.

Pulse and weight


Temperature and weight
Pulse and blood pressure
Temperature and blood pressure

220. A nurse is caring for a group of adult clients on an acute care


medical-surgical nursing unit. The nurse understands that which
of the following clients would be the least likely candidate for
parenteral nutrition (PN)?
a. A 66-year-old client with extensive burns
b. A 42-year-old client who has had an open
cholecystectomy
c. A 27-year-old client with severe exacerbation of Crohn's
disease
d. A 35-year-old client with persistent nausea and vomiting from
chemotherapy
221. The physician orders fat emulsion (Intralipids), given
intravenously, for a client. The nurse should consult with the
physician before administering the fat emulsion solution if which
of the following is noted in the client's record?
a.
b.
c.
d.

The client is receiving parenteral nutrition (PN).


The client has an allergy to iodine.
The client has a blood glucose level of 120 mg/dL.
The client has an allergy to egg yolks.

222. A client is seen in the health care clinic, and a vitamin K


deficiency is suspected. On assessment of the client, the nurse
would expect to note which of the following if this vitamin
deficiency were present?
a.
b.
c.
d.

Client complaints of night blindness


Signs of clotting problems
Scaly skin
Client complaints of skeletal pain

223. The client is admitted to the hospital with a suspected


diagnosis of Hodgkin's disease. Which assessment finding would
the nurse expect to note specifically in the client?
a. Fatigue
b. Weakness

c. Weight gain
d. Enlarged lymph nodes
224. A client is diagnosed with multiple myeloma and the client
asks the nurse about the diagnosis. The nurse bases the response
on which description of this disorder?
a.
b.
c.
d.

Altered red blood cell production


Altered production of lymph nodes
Malignant exacerbation in the number of leukocytes
Malignant proliferation of plasma cells within the bone

225. A gastrectomy is performed on a client with gastric cancer. In


the immediate postoperative period, the nurse notes bloody
drainage from the nasogastric tube. Which of the following is the
appropriate nursing intervention?
a.
b.
c.
d.

Notify the physician.


Measure abdominal girth.
Irrigate the nasogastric tube.
Continue to monitor the drainage.

226. The nurse is assessing the colostomy of a client who has had
an abdominal perineal resection for a bowel tumor. Which of the
following assessment findings indicates that the colostomy is
beginning to function?
a.
b.
c.
d.

Absent bowel sounds


The passage of flatus
The client's ability to tolerate food
Bloody drainage from the colostomy

227. A postoperative client asks a nurse why it is so important to


deep breathe and cough after surgery. When formulating a
response, the nurse incorporates the understanding that retained
pulmonary secretions in a postoperative client can lead to:
a.
b.
c.
d.

Pneumonia
Fluid imbalance
Pulmonary embolism
Carbon dioxide retention

228. A nurse is developing a plan of care for a client scheduled for


surgery. The nurse should include which activity in the nursing
care plan for the client on the day of surgery?
a. Have the client void immediately before going into
surgery.

b. Avoid oral hygiene and rinsing with mouthwash.


c. Verify that the client has not eaten for the last 24 hours.
d. Report immediately any slight increase in blood pressure or
pulse.
229. A nurse assesses a client's surgical incision for signs of
infection. Which finding by the nurse would be interpreted as a
normal finding at the surgical site?
a.
b.
c.
d.

Red, hard skin


Serous drainage
Purulent drainage
Warm, tender skin

230. A nurse has instructed a preoperative client using an incentive


spirometer to sustain the inhaled breath for 3 seconds. When the
client asks about the rationale for this action, the nurse explains
in simple terms that the primary benefit is to do which of the
following?
a.
b.
c.
d.

Dilate the major bronchi.


Increase surfactant production.
Enhance ciliary action in the tracheobronchial tree.
Maintain inflation of the alveoli.

231. A nursing student needs to administer potassium chloride


intravenously as prescribed to a client with hypokalemia. The
nursing instructor determines that the student is unprepared for
this procedure if the student states that which of the following is
part of the plan for preparation and administration of the
potassium?
a.
b.
c.
d.

Obtaining a controlled intravenous (IV) infusion pump


Monitoring urine output during administration
Preparing the medication for bolus administration
Diluting the medication in appropriate amount of normal
saline

232. Salicylic acid is prescribed for a client with a diagnosis of


psoriasis. The nurse monitors the client, knowing that which of
the following would indicate the presence of systemic toxicity
from this medication?
a.
b.
c.
d.

Tinnitus
Diarrhea
Constipation
Decreased respirations

233. The physician has prescribed silver sulfadiazine (Silvadene) for


the client with a partial-thickness burn, which has cultured
positive for gram-negative bacteria, and the nurse provides
information to the client about the medication. Which statement
made by the client indicates a lack of understanding about the
treatments?
a.
b.
c.
d.

"The medication is an antibacterial."


"The medication will help heal the burn."
"The medication will permanently stain my skin."
"The medication should be applied directly to the wound."

234. Auranofin (Ridaur has been prescribed for a client with


rheumatoid arthritis. The nurse provides instructions to the client
about the medication and tells the client to notify the physician if
which of the following occurs?
a.
b.
c.
d.

Nausea
Loss of appetite
Diarrhea
Metallic taste in the mouth

235. A nurse in the postpartum unit is caring for a client who has
just delivered a newborn infant following a pregnancy with a
placenta previa. The nurse reviews the plan of care and prepares
to monitor the client for which risk associated with placenta
previa?
a.
b.
c.
d.

Infection
Hemorrhage
Chronic hypertension
Disseminated intravascular coagulation

236. A postpartum nurse is providing instructions to a client after


delivery of a healthy infant. The nurse instructs the client that she
should expect normal bowel elimination to return:
a.
b.
c.
d.

3 days postpartum
7 days postpartum
On the day of delivery
Within 2 weeks postpartum

237. A nurse is preparing to perform a fundal assessment on a


postpartum client. The initial nursing action in performing this
assessment is which of the following?
a. Ask the client to turn on her side.
b. Ask the client to urinate and empty her bladder.

c. Massage the fundus gently before determining the level of


the fundus.
d. Ask the client to lie flat on her back with the knees and legs
flat and straight.
238. A postpartum client is diagnosed with cystitis. The nurse plans
for which priority nursing intervention in the care of the client?
a.
b.
c.
d.

Providing sitz baths


Encouraging fluid intake
Placing ice on the perineum
Monitoring hemoglobin and hematocrit levels

239. The client arrives at the emergency department with


complaints of low abdominal pain and hematuria. The client is
afebrile. The nurse next assesses the client to determine a history
of:
a.
b.
c.
d.

Pyelonephritis
Glomerulonephritis
Trauma to the bladder or abdomen
Renal cancer in the client's family

240. A nurse is assessing the patency of a client's left arm


arteriovenous fistula prior to initiating hemodialysis. Which finding
indicates that the fistula is patent?
a.
b.
c.
d.

Palpation of a thrill over the fistula


Presence of a radial pulse in the left wrist
Absence of a bruit on auscultation of the fistula
Capillary refill less than 3 seconds in the nail beds of the
fingers on the left hand

241. The nurse is reviewing the client's record and notes that the
physician has documented that the client has a renal disorder. On
review of the laboratory results, the nurse most likely would
expect to note which of the following?
a.
b.
c.
d.

Decreased hemoglobin level


Elevated creatinine level
Decreased red blood cell count
Decreased white blood cell count

242. A nurse is caring for a client with acute renal failure (ARF).
When performing an assessment, the nurse would expect to note
which of the following breathing patterns?
a. Decreased respirations

b. Apnea
c. Cheyne-Stokes respirations
d. Kussmaul's respirations
243. An emergency department nurse is assessing a client who has
sustained a blunt injury to the chest wall. Which of these signs
would indicate the presence of a pneumothorax in this client?
a.
b.
c.
d.

A low respiratory rate


Diminished breath sounds
The presence of a barrel chest
A sucking sound at the site of injury

244. A nurse is suctioning fluids from a client via a tracheostomy


tube. When suctioning, the nurse must limit the suctioning time to
a maximum of:
a.
b.
c.
d.

1 minute
5 seconds
10 seconds
30 seconds

245. A client has experienced pulmonary embolism. A nurse


assesses for which symptom, which is most commonly reported?
a.
b.
c.
d.

Hot, flushed feeling


Sudden chills and fever
Chest pain that occurs suddenly
Dyspnea when deep breaths are taken

246. A nurse is instructing a hospitalized client with a diagnosis of


emphysema about measures that will enhance the effectiveness
of breathing during dyspneic periods. Which of the following
positions will the nurse instruct the client to assume?
a.
b.
c.
d.

Sitting up in bed
Side-lying in bed
Sitting in a recliner chair
Sitting on the side of the bed and leaning on an
overbed table

247. A nurse is preparing to change the parenteral nutrition (PN)


solution bag and tubing. The client's central venous line is located
in the right subclavian vein. The nurse asks the client to take
which essential action during the tubing change?
a. Breathe normally.
b. Turn the head to the right.

c. Exhale slowly and evenly.


d. Take a deep breath, hold it, and bear down.
248. A nurse is caring for a restless client who is beginning
nutritional therapy with parenteral nutrition (PN). The nurse
should plan to ensure that which of the following is done to
prevent the client from injury?
a.
b.
c.
d.

Calculate daily intake and output.


Monitor the temperature once daily.
Secure all connections in the PN system.
Monitor blood glucose levels every 12 hours.

249. A nurse develops a plan of care for a client with deep vein
thrombosis. Which client position or activity in the plan will be
included?
a.
b.
c.
d.

Out-of-bed activities as desired


Bed rest with the affected extremity kept flat
Bed rest with elevation of the affected extremity
Bed rest with the affected extremity in a dependent position

250. Of the following infection control methods, which would most


effectively prevent hepatitis B?
a.
b.
c.
d.

Hand-washing daily
Hepatitis B (HBV) vaccine
Proper personal hygiene
Immune globulin

251. A nurse is preparing to insert a nasogastric tube into a client.


The nurse places the client in which position for insertion?
a) Right side
b) Low Fowler's
c) High Fowler's
d) Supine with the head flat
252. A nurse is suctioning fluids from a client via a tracheostomy
tube. When suctioning, the nurse must limit the suctioning time to
a maximum of:
a) 1 minute
b) 5 seconds
c) 10 seconds

d) 30 seconds
253. An oxygen delivery system is prescribed for a client with
chronic obstructive pulmonary disease to deliver a precise oxygen
concentration. Which oxygen delivery system would the nurse
anticipate to be prescribed?
a) Face tent
b) Venturi mask
c) Aerosol mask
d) Tracheostomy collar
254. A client has experienced pulmonary embolism. A nurse
assesses for which symptom, which is most commonly reported?
a) Hot, flushed feeling
b) Sudden chills and fever
c) Dyspnea when deep breaths are taken
d) Chest pain that occurs suddenly
255. A nurse is caring for a hospitalized client who is retaining
carbon dioxide (CO2) because of respiratory disease. The nurse
plans care anticipating that as the clients CO2 level rises, the pH
should:
a) Rise
b) Fall
c) Remain unchanged
d) Double
256. A nurse is assessing the patency of a clients left arm
arteriovenous fistula prior to initiating hemodialysis. Which finding
indicates that the fistula is patent?
a) Palpation of a thrill over the fistula
b) Presence of a radial pulse in the left wrist
c) Absence of a bruit on auscultation of the fistula
d) Capillary refill less than 3 seconds in the nail beds of the
fingers on the left hand
257. A nurse in the postpartum unit is caring for a client who has
just delivered a newborn infant following a pregnancy with a
placenta previa. The nurse reviews the plan of care and prepares
to monitor the client for which risk associated with placenta
previa?
a) Infection
b) Hemorrhage
c) Chronic hypertension

d) Disseminated intravascular coagulation


258. A nurse is planning care for a postpartum client who had a
vaginal delivery 2 hours ago. The client had a midline episiotomy
and has several hemorrhoids. What is the priority nursing
diagnosis for this client?
a) Acute pain
b) Disturbed body image
c) Impaired urinary elimination
d) Risk for imbalanced fluid volume
259. A nurse is teaching a postpartum client about breast-feeding.
Which of the following instructions should the nurse include?
a) The diet should include additional fluids
b) Prenatal vitamins should be discontinued
c) Soap should be used to cleanse the breasts
d) Birth control measures are unnecessary while breastfeeding
260. A nursing student needs to administer potassium chloride
intravenously as prescribed to a client with hypokalemia. The
nursing instructor determines that the student is unprepared for
this procedure if the student states that which of the following is
part of the plan for preparation and administration of the
potassium?
a) Obtaining a controlled intravenous (IV) infusion pump
b) Monitoring urine output during administration
c) Preparing the medication for bolus administration
d) Diluting the medication in appropriate amount of normal
saline
261. A home care nurse visits a client recently diagnosed with
diabetes mellitus who is taking Humulin NPH insulin daily. The
client asks the nurse how to store the unopened vials of insulin.
The nurse tells the client to:
a) Freeze the insulin
b) Refrigerate the insulin
c) Store the insulin in a dark, dry place
d) Keep the insulin at room temperature
262. A postoperative client asks a nurse why it is so important to
deep breathe and cough after surgery. When formulating a
response, the nurse incorporates the understanding that retained
pulmonary secretions in a postoperative client can lead to:

a) Pneumonia
b) Fluid imbalance
c) Pulmonary embolism
d) Carbon dioxide retention
263. A nurse is monitoring the status of a postoperative client. The
nurse would become most concerned with which of the following
signs that could indicate an evolving complication?
a) Increasing restlessness
b) A pulse of 86 beats/min
c) Blood pressure of 110/70 mm Hg
d) Hypoactive bowel sounds in all four quadrants
264. A nurse is providing home care instructions to the parents of
an infant who had a surgical repair of an inguinal hernia. The
nurse instructs the parents to do which of the following to prevent
infection at the surgical site?
a) Change the diapers as soon as they become damp
b) Report a fever immediately
c) Soak the infant in a tub bath twice a day for the next 5 days
d) Restrict the infant's physical activity
265. The nurse is preparing a preoperative client for transfer to the
operating room. The nurse should take which of the following
actions in the care of this client at this time?
a) Ensure that the client has voided
b) Administer all the daily medications
c) Have the client practice postoperative breathing exercises
d) Verify that the client has not eaten for the last 24 hours
266. The client is admitted to the hospital with a suspected
diagnosis of Hodgkins disease. Which assessment finding would
the nurse expect to note specifically in the client?
a) Fatigue
b) Weakness
c) Weight gain
d) Enlarged lymph nodes
267. The nurse is assessing the colostomy of a client who has had
an abdominal perineal resection for a bowel tumor. Which of the
following assessment findings indicates that the colostomy is
beginning to function?

a) Absent bowel sounds


b) The passage of flatus
c) The clients ability to tolerate food
d) Bloody drainage from the colostomy
268. A nurse has admitted a client to the clinical nursing unit after
a modified right radical mastectomy for the treatment of breast
cancer. The nurse plans to place the right arm in which of the
following positions?
a) Elevated above shoulder level
b) Level with the right atrium
c) Dependent to the right atrium
d) Elevated on a pillow
269. A nurse is reviewing the progress notes for a client admitted
to the nursing unit with a suspected diagnosis of leukemia. The
nurse notes that the diagnosis of leukemia has been confirmed.
The nurse interprets that results have been reported to the
physician for which of the following diagnostic tests?
a) Complete blood cell count
b) Platelet count
c) White blood cell count
d) Bone marrow biopsy
270. The nurse is teaching a client who has iron deficiency anemia
about foods she should include in her diet. The nurse determines
that the client understands the dietary modifications if she selects
which of the following from her menu?
a) Nuts and milk
b) Coffee and tea
c) Cooked rolled oats and fish
d) Oranges and dark green leafy vegetables
271. The nurse is conducting a dietary assessment on a client who
is on a vegan diet. The nurse provides dietary teaching focusing
on foods high in which vitamin that may be lacking in a vegan
diet?
a) Vitamin A
b) Vitamin B12
c) Vitamin C
d) Vitamin E

272. A nurse is assigned to care for an 8-year-old child with a


diagnosis of a basilar skull fracture. The nurse reviews the
physicians prescriptions and contacts the physician to question
which prescription?
a) Suction as needed
b) Obtain daily weight
c) Provide clear liquid intake
d) Maintain a patent intravenous line
273. A child is brought to the hospital emergency department for
an injury to the lower right arm that occurred in a fall off a bicycle.
On assessment, the nurse notes that the skin at the site of the
injury is intact. A fracture is suspected, and a radiograph is taken.
The nurse can see on the radiograph viewer that the fracture of
the bone is across the entire bone shaft with some possible
displacement. The nurse determines that this childs fracture is a:
a) Simple fracture
b) Compound fracture
c) Greenstick fracture
d) Comminuted fracture
274. A nurse is conducting health screening for osteoporosis. Which
of the following clients is at greatest risk of developing this
disorder?
a) A 25-year-old woman who jogs
b) A 36-year-old man who has asthma
c) A 70-year-old man who consumes excess alcohol
d) A sedentary 65-year-old woman who smokes
cigarettes
275. A community health nurse is providing an educational session
for community members regarding dietary measures that will
assist in reducing the risk of osteoporosis. The nurse instructs the
community members to increase dietary intake of which food
known to be helpful in minimizing this risk?
a) Turkey
b) Yogurt
c) Spaghetti
d) Shellfish
276. A child has fluid volume deficit. The nurse performs an
assessment and determines that the child is improving and the
deficit is resolving if:

a) The child has no tears


b) Urine specific gravity is 1.030
c) Urine output is less than 1 mL/kg/hr
d) Capillary refill is less than 2 seconds
277. A clinic nurse is assessing a child for dehydration. The nurse
determines that the child is moderately dehydrated if which of the
following is noted on assessment?
a) Oliguria
b) Flat fontanels
c) Pale skin color
d) Moist mucous membranes
278. The home care nurse is visiting a client whose husband died 6
months ago. Which behavior by the client indicates ineffective
coping?
a) Neglecting her personal grooming
b) Looking at old snapshots of her family
c) Participating in a senior citizens program
d) Visiting her husband s grave once a month
279. A nurse is caring for a client with a phobia who is being
treated for the condition. The client is introduced to short periods
of exposure to the phobic object while in a relaxed state. The
nurse understands that this form of behavior modification can
best be described as:
a) Milieu therapy
b) Aversion therapy
c) Self-control therapy
d) Systematic desensitization
280. A client is admitted to a medical nursing unit with a diagnosis
of acute blindness. Many tests are performed, and there seems to
be no organic reason why this client cannot see. The client
became blind after witnessing a hit-and-run car accident, when a
family of three was killed. A nurse suspects that the client may be
experiencing a:
a) Psychosis
b) Repression
c) Conversion disorder
d) Dissociative disorder
281. The nurse has obtained a unit of blood from the blood bank
and has checked the blood bag properly with another nurse. Just

before beginning the transfusion, the nurse assesses which


priority item?
a) Vital signs
b) Skin color
c) Urine output
d) Latest hematocrit level
282. A pediatric client with ventricular septal defect repair is placed
on a maintenance dosage of digoxin (Lanoxin) elixir. The dosage
is 0.07 mg/kg/day, and the clients weight is 7.2 kg. The physician
prescribes the digoxin to be given twice daily. A nurse prepares
how much digoxin to administer to the client at each dose?
a) 0.5 mg
b) 2.5 mg
c) 0.25 mg
d) 0.37 mg
283. A nurse has an order to transfuse a unit of packed red blood
cells to a client who does not currently have an intravenous (IV)
line inserted. When obtaining supplies to start the IV infusion, the
nurse selects an angiocatheter with a size of:
a) 18 gauge
b) 21 gauge
c) 22 gauge
d) 24 gauge
284. A client is in the bathroom when the nurse arrives at his room
with his scheduled medications. The client calls to the nurse, "Just
leave my medication on the bedside table like the rest of the
nurses, and I will take it when I get finished." What is the nurse's
best action?
a) Leave the medication at the bedside as the client requested
b) Tell the client you will be back when he is finished
c) Tell the unlicensed assistive personnel (UAP) to give it to
the client when he is finished
d) Let another nurse who is not busy give the client his
medication when he is finished
285. An adult female client has a hemoglobin level of 10.8 g/dL.
The nurse interprets that this result is most likely caused by which
of the following conditions noted in the clients history?
a) Dehydration
b) Heart failure

c) Iron deficiency anemia


d) Chronic obstructive pulmonary disease
286. Ferrous sulfate (iron) has been prescribed for the pregnant
client. Before initiating the therapy, the nurse reviews which of
the following laboratory values that reflects the need for this
dietary supplement?
a) Hemoglobin level
b) Prothrombin time
c) Bleeding time
d) Clotting time
287. A nurse is caring for a client in labor. The nurse determines
that the client is beginning the second stage of labor when which
of the following assessments is noted?
a) The contractions are regular
b) The membranes have ruptured
c) The cervix is dilated completely
d) The client begins to expel clear vaginal fluid
288. A nurse assists in the vaginal delivery of a newborn infant.
After the delivery, the nurse observes the umbilical cord lengthen
and a spurt of blood from the vagina. The nurse documents these
observations as signs of:
a) Hematoma
b) Uterine atony
c) Placenta previa
d) Placental separation
289. When assessing a lesion diagnosed as malignant melanoma,
the nurse most likely expects to note which of the following?
a) An irregularly shaped lesion
b) A small papule with a dry, rough scale
c) A firm, nodular lesion topped with crust
d) A pearly papule with a central crater and a waxy border
290. The evening nurse reviews the nursing documentation in the
clients chart and notes that the day nurse has documented that
the client has a stage II pressure ulcer in the sacral area. Which of
the following would the nurse expect to note on assessment of
the clients sacral area?
a) Intact skin
b) Full-thickness skin loss

c) Exposed bone, tendon, or muscle


d) Partial-thickness skin loss of the dermis
291. The nurse is caring for a client following an autograft and
grafting to a burn wound on the right knee. Which of the following
would the nurse anticipate to be prescribed for the client?
a) Out of bed
b) Bathroom privileges
c) Immobilization of the affected leg
d) Placing the affected leg in a dependent position
292. An emergency department nurse is caring for a client who has
sustained chemical burns to the esophagus after ingestion of lye.
The nurse reviews the physician's prescriptions and plans to
question a prescription for which of the following?
a) Nothing by mouth (NPO) status
b) Gastric lavage
c) Intravenous (IV) fluid therapy
d) Preparation for laboratory studies
293. A 6-month-old infant receives a diphtheria, tetanus, and
acellular pertussis (DTaP) immunization at a well-baby clinic. The
mother returns home and calls the clinic to report that the infant
has developed swelling and redness at the site of injection. A
nurse tells the mother to:
a) Monitor the infant for a fever
b) Bring the infant back to the clinic
c) Apply a hot pack to the injection site
d) Apply an ice pack to the injection site
294. The mother of a preschooler who attends day care calls a
clinic nurse and tells the nurse that the child is constantly
scratching the perianal area and that the area is irritated. The
nurse suspects the possibility of pinworm infection (enterobiasis).
The nurse instructs the mother to obtain a rectal specimen by a
tape test and tells the mother to obtain the specimen:
a) After bathing
b) After toileting
c) When the child is put to bed
d) In the morning, when the child awakens
295. At what age should the child receive the first dose of the
measles, mumps, and rubella vaccine?

a) 2 years
b) 4 years
c) 12 months
d) 22 months
296. The nurse prepares to give a bath and change the bed linens
of a client with cutaneous Kaposis sarcoma lesions. The lesions
are open and draining a scant amount of serous fluid. Which of
the following would the nurse incorporate into the plan during the
bathing of this client?
a) Wearing gloves
b) Wearing a gown and gloves
c) Wearing a gown, gloves, and a mask
d) Wearing a gown and gloves to change the bed linens and
gloves only for the bath
297. A client is suspected of having systemic lupus erythematous.
The nurse monitors the client, knowing that which of the following
is one of the initial characteristic signs of systemic lupus
erythematous?
a) Weight gain
b) Subnormal temperature
c) Elevated red blood cell count
d) Rash on the face across the bridge of the nose and
on the cheeks
298. The client with acquired immunodeficiency syndrome has
begun therapy with zidovudine (Retrovir, azidothymidine, AZT,
ZDV). The nurse carefully monitors which of the following
laboratory results during treatment with this medication?
a) Blood culture
b) Blood glucose level
c) Complete blood count
d) Blood urea nitrogen level
299. A nurse is providing home care instructions to the mother of a
10-year-old child with hemophilia. Which of the following activities
should the nurse suggest that the child could participate in safely
with peers?
a) Soccer
b) Basketball
c) Swimming
d) Field hockey

300. A nurse is monitoring a 3-year-old child for signs and


symptoms of increased intracranial pressure (ICP) after a
craniotomy. The nurse plans to monitor for which early sign or
symptom of increased ICP?
a) Excessive vomiting
b) Bulging anterior fontanel
c) Increasing head circumference
d) Complaints of a frontal headache
301. A 4-year-old child is admitted to the hospital for abdominal
pain. The mother reports that the child has been pale and
excessively tired and is bruising easily. On physical examination,
lymphadenopathy and hepatosplenomegaly are noted. Diagnostic
studies are being performed on the child because acute
lymphocytic leukemia is suspected. The nurse understands that
which diagnostic study would confirm this diagnosis?
a) Platelet count
b) Lumbar puncture
c) Bone marrow biopsy
d) White blood cell count
302. Oral iron supplements are prescribed for a 6-year-old child
with iron deficiency anemia (IDA). The nurse instructs the mother
to administer the iron with which of the following food items?
a) Water
b) Milk
c) Apple juice
d) Orange juice
303. A client with a diagnosis of asthma is admitted to the hospital
with respiratory distress. What type of adventitious lung sounds
would the nurse expect to hear when performing a respiratory
assessment on this client?
a) Stridor
b) Crackles
c) Wheezes
d) Diminished
304. The nurse is instructing a client how to perform a testicular
self-examination (TSE). The nurse explains that the best time to
perform this exam is:
a) After a shower or bath
b) While standing to void

c) After having a bowel movement


d) While lying in bed before arising
305. The community health nurse is instructing a group of female
clients about breast self-examination. The nurse instructs the
clients to perform the examination:
a) At the onset of menstruation
b) Every month during ovulation
c) Weekly at the same time of day
d) 1 week after menstruation begins
306. A nurse has obtained a personal and family history from a
client with a neurological disorder. Which of the following factors
in the clients history is not associated with added risk for
neurological problems?
a) Previous back injury
b) Allergy to pollen
c) History of hypertension
d) History of headaches
307. The nurse is planning to teach the client with
gastroesophageal reflux disease about substances that will
increase the lower esophageal sphincter pressure. Which item
should the nurse include on this list?
a) Coffee
b) Chocolate
c) Fatty foods
d) Nonfat milk
308. The physician has determined that the client with hepatitis
has contracted the infection from contaminated food. The nurse
understands that this client is most likely experiencing what type
of hepatitis?
a.
b.
c.
d.

Hepatitis A
Hepatitis B
Hepatitis C
Hepatitis D

309. The nurse is caring for a client with a diagnosis of chronic


gastritis. The nurse monitors the client knowing that this client is
at risk for which vitamin deficiency?
a) Vitamin A
b) Vitamin B12

c) Vitamin C
d) Vitamin E
310. A child is hospitalized because of persistent vomiting. The
nurse monitors the child closely for:
a) Diarrhea
b) Metabolic acidosis
c) Metabolic alkalosis
d) Hyperactive bowel sounds
311. A clinic nurse is assessing jaundice in a child with hepatitis.
Which anatomical area would provide the best data regarding the
presence of jaundice?
a) The nail beds
b) The skin in the abdominal area
c) The skin in the sacral area
d) The membranes in the ear canal
312. A nurse is assigned to care for a group of clients. On review of
the clients medical records, the nurse determines that which
client is at risk for a fluid volume deficit?
a) A client with a colostomy
b) A client with congestive heart failure
c) A client on long-term corticosteroid therapy
d) A client receiving frequent wound irrigations
313. A nurse is performing an assessment on a client admitted to
the hospital with a diagnosis of dehydration. Which of the
following assessment findings should the nurse expect to note?
a) Bradycardia
b) Changes in mental status
c) Bilateral crackles in the lungs
d) Elevated blood pressure
314. A woman was working in her garden. She accidentally sprayed
insecticide into her right eye. She calls the emergency
department frantic and screaming for help. The nurse should
instruct the woman to take which immediate action?
a) Call the physician
b) Come to the emergency room
c) Irrigate the eyes with water
d) Irrigate the eyes with diluted hydrogen peroxide

315. A day care nurse is observing a 2-year-old child and suspects


that the child may have strabismus. Which observation made by
the nurse might indicate this condition?
a) The child has difficulty hearing
b) The child consistently tilts the head to see
c) The child consistently turns the head to see
d) The child does not respond when spoken to
316. After a tonsillectomy, a nurse reviews the physicians
postoperative prescriptions. Which of the following physicians
prescriptions does the nurse question?
a) Monitor for bleeding
b) Suction every 2 hours
c) Give no milk or milk products
d) Give clear, cool liquids when awake and alert
317. A nurse hears a client calling out for help, hurries down the
hallway to the clients room, and finds the client lying on the floor.
The nurse performs a thorough assessment, assists the client
back to bed, notifies the physician of the incident, and completes
an incident report. Which of the following should the nurse
document on the incident report?
a) The client fell out of bed
b) The client climbed over the side rails
c) The client was found lying on the floor
d) The client became restless and tried to get out of bed
318. During a home visit, the nurse discusses health care concerns
with the client and establishes mutual goals to help the client
become more independent. In this role, the nurse is functioning
as:
a) A researcher
b) A resource linker
c) A collaborator
d) An advocate
319. After several diagnostic tests, a client is diagnosed with
diabetes insipidus. A nurse performs an assessment on the client,
knowing that which symptom is most indicative of this disorder?
a) Fatigue
b) Diarrhea
c) Polydipsia
d) Weight gain

320. A client who suffered a severe head injury has had vigorous
treatment to control cerebral edema. Brain death has been
determined. The nurse prepares to carry out which of the
following measures to maintain viability of the kidneys before
organ donation?
a) Monitoring temperature
b) Administering intravenous (IV) fluids
c) Assessing lung sounds
d) Performing range-of-motion exercises to the extremities
321. The client is diagnosed with a disorder involving the inner ear.
Which of the following is the most common client complaint
associated with a disorder involving this part of the ear?
a) Pruritus
b) Tinnitus
c) Hearing loss
d) Burning in the ear
322. A nursing instructor asks a nursing student to present a
clinical conference to peers regarding Freud's psychosexual
stages of development, specifically the anal stage. The student
plans the conference, knowing that which of the following most
appropriately relates to this stage of development?
a) This stage is associated with toilet training
b) This stage is characterized by the gratification of self
c) This stage is characterized by a tapering off of conscious
biological and sexual urges
d) This stage is associated with pleasurable and conflicting
feelings about the genital organs
323. A nurse is assigned to care for four clients. In planning client
rounds, which client should the nurse assess first?
a) A client scheduled for a chest x-ray
b) A client requiring daily dressing changes
c) A postoperative client preparing for discharge
d) A client receiving nasal oxygen who had difficulty
breathing during the previous shift
324. A client receiving parenteral nutrition (PN) complains of a
headache. A nurse notes that the client has an increased blood
pressure, bounding pulse, jugular vein distention, and crackles
bilaterally. The nurse determines that the client is experiencing
which complication of PN therapy?

a) Sepsis
b) Air embolism
c) Hypervolemia
d) Hyperglycemia
325. A nurse is performing cardiopulmonary resuscitation (CPR) on
an infant. When performing chest compressions, the nurse
compresses at least:
a) 60 times per minute
b) 80 times per minute
c) 100 times per minute
d) 160 times per minute

326. A female client who has had a myocardial infarction asks the nurse
why she should not bear down or strain to ensure having a bowel
movement. The nurse incorporates in a response that this would
trigger: Options:
a. Vagus nerve stimulation, causing a decrease in heart
rate and cardiac contractility .
b. Vagus nerve stimulation, causing an increase in heart rate and
cardiac contractility
c. Sympathetic nerve stimulation, causing an increase in heart
rate and cardiac contractility
d. Sympathetic nerve stimulation, causing a decrease in heart
rate and cardiac contractility

327. A client is experiencing a decrease in cardiac output. The


nurse understands that the client could benefit from greater
endogenous production of which of the following substances,
which also increases blood pressure?
Options:
a.
b.
c.
d.

Epinephrine
Norepinephrine
Dopamine
Serotonin

328. A new nursing graduate is caring for a client who is attached


to a cardiac monitor. While assisting the client with bathing, the
nurse notes the sudden development of ventricular tachycardia
(VT), but the client remains alert and oriented and has a pulse.
Select the interventions that the nurse would take. (Select all that
apply.)
Options:
a. Defibrillate the client.

b.
c.
d.
e.
f.

Assess airway, breathing, and circulation.


Initiate cardiopulmonary resuscitation (CPR).
Administer oxygen.
Obtain an electrocardiogram (ECG).
Contact the physician.
Answer:
b.d.e.f

329. Which of the following laboratory test results may be


associated with peaked or tall, tented T waves on a client s
electrocardiogram (ECG)?
Options:
a.
b.
c.
d.

Chloride level of 98 mEq/L


Sodium level of 135 mEq/L
Potassium level of 6.8 mEq/L
Magnesium level of 1.6 mEq/

330. A nurse is assessing a client with a history of cardiac problems.


Where should the nurse place the stethoscope in order to hear the first
heart sound (S 1 ) the loudest?
Options:
a. Over the second intercostal space at the left sternal border
b. Over the fourth intercostal space at the right sternal border
c. Over the second intercostal space at the right sternal border
d. Over the fifth intercostal space in the left midclavicular
line
331. A nurse providing care for a client on cardiac telemetry notes this
cardiac rhythm on the monitor. The nurse interprets the rhythm as:
Options:
a. Normal sinus rhythm (NSR)
b. Bradycardia
c. Atrial fibrillation
d. Tachycardia
332. A nurse is assigned the care of a client who experienced a
myocardial infarction and is being monitored by cardiac telemetry. The
nurse notes the sudden onset of this cardiac rhythm on the monitor. The
nurse immediately:
Options:
a. Takes the client s blood pressure
b. Initiates cardiopulmonary resuscitation (CPR)
c. Places a nitroglycerin tablet under the client s tongue
d. Continues to monitor the client for 1 minute and then contacts the
physician
333. A nurse is assigned the care of a client with a diagnosis of
congestive heart failure who is receiving intravenous doses of
furosemide (Lasix). The client is attached to cardiac telemetry, and the
nurse is monitoring the client s cardiac status. The nurse notes that the

client s cardiac rhythm has changed to this pattern. The nurse


determines that the most likely cause of this cardiac rhythm in this
client is: Options:
a. The effectiveness of the furosemide
b. The presence of hypokalemia
c. Pacemaker dysfunction
d. An impending myocardial infarction (MI)
334. A client is attached to a cardiac monitor, and the nurse notes the
presence of this cardiac rhythm on the monitor. The nurse quickly
assesses the client, knowing that this rhythm is indicative of:
Options:
a. Atrial fibrillation
b. Premature ventricular complexes
c. Ventricular tachycardia (VT)
d. Ventricular fibrillation (VF)
335. A nurse is assessing a client s legs for the presence of edema. The
nurse notes that the client has mild pitting with slight indentation and
no perceptible swelling of the leg. The nurse documents this finding as
defining:
Options:
a. 1 + edema
b. 2 + edema
c. 3 + edema
d. 4 + edema
336. A client is diagnosed with iron deficiency anemia, and ferrous
sulfate (Feosol) is prescribed. The nurse tells the client that it is best to
take the medication with:
Options:
a. Milk
b. Boiled
c. Tomato juice
d. Pineapple juice
337. A nurse is teaching a client with cardiomyopathy about home care
safety measures. The nurse addresses which of the following as the
most important measure to ensure client safety?
Options:
a. Assessing pain
b. Avoiding over-the-counter medications
c. Administering vasodilators
d. Moving slowly from a sitting to a standing position
338. A client with an acute respiratory infection is admitted to the
hospital with a diagnosis of sinus tachycardia. In developing a plan of
care for the client, the nurse includes which of the following? Options:
a. Providing the client with short, frequent walks
b. Measuring the client s pulse each shift
c. Eliminating sources of caffeine from meal trays
d. Limiting oral and intravenous fluids

339. A client is scheduled for elective cardioversion to treat chronic highrate atrial fibrillation. The nurse determines that the client is not yet
ready for the procedure after noting which of the following? Options:
a. The client s digoxin (Lanoxin) has been withheld for the last
48 hours .
b. The client has received a dose of midazolam (Versed)
intravenously.
c. The client is wearing a nasal cannula delivering
oxygen at 2 L/min .
d. The defibrillator has the synchronizer turned on and is set at
50 joules (J).
340. A nurse is assisting in the care of a client scheduled for
cardioversion. The nurse plans to set the defibrillator to which of the
following starting energy range levels, depending on the specific
physician order?
Options:
a. 50 to 100 joules
b. 150 to 200 joules
c. 250 to 300 joules
d. 350 to 400 joules
341. A client has developed atrial fibrillation with a ventricular rate of
150 beats/min . The nurse assesses the client for:
Options:
a. Hypotension and dizziness
b. Nausea and vomiting
c. Hypertension and headache
d. Flat neck veins

342. A nurse has provided self-care activity instructions to a client after


insertion of an automatic internal cardioverter-defibrillator (AICD). The
nurse determines that further instruction is needed if the client makes
which of the following statements?
Options:
a. I should try to avoid doing strenuous things that would make
my heart rate go up to or above the rate cutoff on the AICD.
b. I should keep away from electromagnetic sources such as
transformers, large electrical generators, metal detectors,
and leaning over running motors.
c. I can perform activities such as swimming, driving, or
operating heavy equipment as I need to.
d. I need to avoid doing anything that could involve rough
contact with the AICD insertion site.
343. A client with a history of hypertension has been prescribed
triamterene (Dyrenium). The nurse determines that the client

understands the impact of this medication on the diet if the client states
to avoid which of the following fruits?
Options:
a. Apples
b. Pears
c. Bananas
d. . Cranberries
344. The health care provider prescribes bedrest for a client in whom a
deep vein thrombosis develops after surgery. From the following list,
select all appropriate nursing interventions to include in this client s
plan of care.
Options:
a. Place in Fowler s position for eating.
b. Encourage increased oral intake of water daily.
c. Encourage coughing with deep breathing.
d. Place thigh-length elastic stockings on the client.
e. Encourage the intake of dark, green leafy vegetables.
e. Place sequential compression boots on the client.
Answer:
b.c.d.
345. Spironolactone (Aldactone), a diuretic, is prescribed for a client with
congestive heart failure. In providing dietary instructions to the client,
the nurse identifies the need to avoid foods that are high in which
electrolyte?
Options:
a. Calcium
b. Potassium
c. Magnesium
d. Phosphorus
346. A client is seen in the urgent care center for complaints of chest
pain that began 3 days earlier. Since that time the client has not been
feeling well and fatigues easily. The nurse would suspect myocardial
infarction at the time of chest pain if a test for which of the following
isoenzymes for lactate dehydrogenase (LDH) came back positive?
Options:
a. LDH 1
b. LDH 3
c. LDH 4
d. LDH 5
347. In planning a low-sodium diet for the client who has recently been
diagnosed with congestive heart failure, the nurse should ask the client
if she would like to have which food item?
Options:
a. Chicken breast
b. Cottage cheese
c. Grilled cheese
d. Beef bouillon

348. A clinic nurse is performing a cardiovascular assessment on a client


and auscultates the chest over the apex of the heart. (Click on sound
icon.) The nurse identifies this sound as which of the following? Options:
a. First heart sound , S 1
b. . Ventricular gallop
c. Third heart sound , S 3
d. Fourth heart sound , S4
349. Cardiac monitoring leads are placed on a client who is at risk for
premature ventricular contractions (PVCs). The nurse assesses the
client s heart rhythm to detect PVCs by looking for:
Options:
a. Premature beats followed by a compensatory pause
b. QRS complexes that are short and narrow
c. Inverted P waves before the QRS complexes
d. A P wave preceding every QRS complex
350. A nurse is assisting in positioning the client for pericardiocentesis to
treat cardiac tamponade. The best position for this client is:
Options:
a. Lying on the left side with a pillow under the chest wall
b. Lying on the right side with a pillow under the head
c. Supine with the head of bed elevated at a 45- to 60degree angle
d. Supine with slight Trendelenburg position
351. A nurse is assessing a client hospitalized with acute pericarditis. The
nurse monitors the client for cardiac tamponade, knowing that which of
the following is unassociated with this complication of pericarditis?
Options:
a. Pulsus paradoxus
b. Distant heart sounds
c. Distended jugular veins
d. Bradycardia
352. A nurse inquires about smoking history while conducting a hospital
admission assessment for a client with coronary artery disease (CAD).
The most important element of the smoking history for this assessment
is the:
Options:
a. Number of pack-years
b. Brand of cigarettes used
c. Desire to quit smoking
d. Number of past attempts to quit smoking
353. A 52-year- old male client is seen in the physician s office for a
physical examination after experiencing unusual fatigue over the last
several weeks . The client s height is 5 feet 8 inches and his weight is
220 pounds . Vital signs are as follows : temperature , 98.6 F orally ;
pulse , 86 beats/min ; and respirations , 18 breaths/min . The blood
pressure reading is 184/100 mm Hg . A random blood glucose level is
122 mg/dL. Which of the following questions should the nurse ask the
client first?
Options:
a. Do you exercise regularly?

b. Are you considering trying to lose weight?


c. Is there a history of diabetes mellitus in your family?
d. When was the last time you had your blood pressure
checked?
354. A nurse is developing a plan of care for a client with pulmonary
edema. The nurse establishes a goal to have the client participate in
activities that reduce cardiac workload. The nurse identifies which client
action as contributing to this goal?
Options:
a. Elevating the legs when in bed
b. Sleeping in the supine position
c. Using seasonings to improve the taste of food
d. Using a bedside commode
355. A nurse is performing an admission assessment on a client with a
diagnosis of Raynaud s disease. The nurse assesses for associated
signs and symptoms by:
Options:
a. Observing for softening of the nails or nail beds
b. Palpating for diminished or absent peripheral pulses
c. Checking for a rash on the digits
d. Palpating for a rapid or irregular peripheral pulse

356. A patient tells you that her urine is starting to look discolored.
If you believe this change is due to medication, which of the
following patient's medication does not cause urine discoloration?
A. Sulfasalazine
B. Levodopa
C. Phenolphthalein
D. Aspirin
357. You are responsible for reviewing the nursing unit's
refrigerator. If you found the following drug in the refrigerator it
should be removed from the refrigerator's contents?
A. Corgard
B. Humulin (injection)
C. Urokinase
D. Epogen (injection)
358. A 34 year old female has recently been diagnosed with an
autoimmune disease. She has also recently discovered that she is
pregnant. Which of the following is the only immunoglobulin that
will provide protection to the fetus in the womb?
A. IgA
B. IgD
C. IgE
D. IgG
359. A second year nursing student has just suffered a needlestick
while working with a patient that is positive for AIDS. Which of the

following is the most important action that nursing student should


take?
A. Immediately see a social worker
B. Start prophylactic AZT treatment
C. Start prophylactic Pentamide treatment
D. Seek counseling
360. A thirty five year old male has been an insulin-dependent
diabetic for five years and now is unable to urinate. Which of the
following would you most likely suspect?
A. Atherosclerosis
B. Diabetic nephropathy
C. Autonomic neuropathy
D. Somatic neuropathy
361. You are taking the history of a 14 year old girl who has a (BMI)
of 18. The girl reports inability to eat, induced vomiting and
severe constipation. Which of the following would you most likely
suspect?
A. Multiple sclerosis
B. Anorexia nervosa
C. Bulimia
D. Systemic sclerosis
362. A 24 year old female is admitted to the ER for confusion. This
patient has a history of a myeloma diagnosis, constipation,
intense abdominal pain, and polyuria. Which of the following
would you most likely suspect?
A. Diverticulosis
B. Hypercalcaemia
C. Hypocalcaemia
D. Irritable bowel syndrome
363. Rho gam is most often used to treat____ mothers that have a
____ infant.
A. RH positive, RH positive
B. RH positive, RH negative
C. RH negative, RH positive
D. RH negative, RH negative
364. A new mother has some questions about (PKU). Which of the
following statements made by a nurse is not correct regarding
PKU?
A. A Guthrie test can check the necessary lab values.
B. The urine has a high concentration of phenylpyruvic acid
C. Mental deficits are often present with PKU.
D. The effects of PKU are reversible.
365. A patient has taken an overdose of aspirin. Which of the
following should a nurse most closely monitor for during acute
management of this patient?
A. Onset of pulmonary edema

B. Metabolic alkalosis
C. Respiratory alkalosis
D. Parkinson's disease type symptoms
366. A fifty-year-old blind and deaf patient has been admitted to
your floor. As the charge nurse your primary responsibility for this
patient is?
A. Let others know about the patient's deficits
B. Communicate with your supervisor your concerns about the
patient's deficits.
C. Continuously update the patient on the social environment.
D. Provide a secure environment for the patient.
367. A patient is getting discharged from a SNF facility. The patient
has a history of severe COPD and PVD. The patient is primarily
concerned about their ability to breath easily. Which of the
following would be the best instruction for this patient?
A. Deep breathing techniques to increase O2 levels.
B. Cough regularly and deeply to clear airway passages.
C. Cough following bronchodilator utilization
D. Decrease CO2 levels by increase oxygen take output during
meals.
368.
A nurse is caring for an infant that has recently been
diagnosed with a congenital heart defect. Which of the following
clinical signs would most likely be present?
A. Slow pulse rate
B. Weight gain
C. Decreased systolic pressure
D. Irregular WBC lab values
369. A mother has recently been informed that her child has
Down's syndrome. You will be assigned to care for the child at
shift change. Which of the following characteristics is not
associated with Down's syndrome?
A. Simian crease
B. Brachycephaly
C. Oily skin
D. Hypotonicity
370. A patient has recently experienced a (MI) within the last 4
hours. Which of the following medications would most like be
administered?
A. Streptokinase
B. Atropine
C. Acetaminophen
D. Coumadin
371. A patient asks a nurse, My doctor recommended I increase
my intake of folic acid. What type of foods contain folic acids?
A. Green vegetables and liver
B. Yellow vegetables and red meat

C. Carrots
D. Milk
372. A nurse is putting together a presentation on meningitis.
Which of the following microorganisms has noted been linked to
meningitis in humans?
A. S. pneumonia
B. H. influenza
C. N. meningitis
D. Cl. difficile
373. A nurse is administering blood to a patient who has a low
hemoglobin count. The patient asks how long to RBC's last in my
body? The correct response is.
A. The life span of RBC is 45 days.
B. The life span of RBC is 60 days.
C. The life span of RBC is 90 days.
D. The life span of RBC is 120 days.
374. A 65 year old man has been admitted to the hospital for
spinal stenosis surgery. When does the discharge training and
planning begin for this patient?
A. Following surgery
B. Upon admit
C. Within 48 hours of discharge
D. Preoperative discussion
375. A child is 5 years old and has been recently admitted into the
hospital. According to Erickson which of the following stages is the
child in?
A. Trust vs. mistrust
B. Initiative vs. guilt
C. Autonomy vs. shame
D. Intimacy vs. isolation
376. A toddler is 16 months old and has been recently admitted
into the hospital. According to Erickson which of the following
stages is the toddler in?
A. Trust vs. mistrust
B. Initiative vs. guilt
C. Autonomy vs. shame
D. Intimacy vs. isolation
377. A young adult is 20 years old and has been recently admitted
into the hospital. According to Erickson which of the following
stages is the adult in?
A. Trust vs. mistrust
B. Initiative vs. guilt
C. Autonomy vs. shame
D. Intimacy vs. isolation
378. A nurse is making rounds taking vital signs. Which of the
following vital signs is abnormal?

A. 11 year old male 90 b.p.m, 22 resp/min., 100/70 mm Hg


B. 13 year old female 105 b.p.m., 22 resp/min., 105/60
mm Hg
C. 5 year old male- 102 b.p.m, 24 resp/min., 90/65 mm Hg
D. 6 year old female- 100 b.p.m., 26 resp/min., 90/70mm Hg
379. When you are taking a patient's history, she tells you she has
been depressed and is dealing with an anxiety disorder. Which of
the following medications would the patient most likely be taking?
A. Elavil
B. Calcitonin
C. Pergolide
D. Verapamil
380. Which of the following conditions would a nurse not
administer erythromycin?
A. Campylobacterial infection
B. Legionnaire's disease
C. Pneumonia
D. Multiple Sclerosis

381. Which of the following is not one of the key steps in the grief
process?
A. Denial
B. Anger
C. Bargaining
D. Rejection
382. Which of the following matches the definition: covering up a
weakness by stressing a desirable or stronger trait?
A. Compensation
B. Projection
C. Rationalization
D. Dysphoria
383. Which of the following waveforms is most commonly found
with light sleepers?
A. Theta
B. Alpha
C. Beta
D. Zeta

384. Which of the following months matches with an infant first


having the ability to sit-up independently?
A. 4 months
B. 6 months
C. 8 months
D. 10 months
385. Object permanence for toddlers develops in this age range?
A. 5-10 months
B. 10-14 months
C. 12-24 months
D. 15-24 months
386. Which of the following matches the definition: attributing of
our own unwanted trait onto another person?
A. Compensation
B. Projection
C. Rationalization
D. Dysphoria
387. Which of the following matches the definition: the justification
of behaviors using reason other than the real reason?
A. Compensation
B. Projection
C. Rationalization
D. Dysphoria
388. Which of the following matches the definition: response to
severe emotion stress resulting in involuntary disturbance of
physical functions?
A. Conversion disorder
B. Depressive reaction
C. Bipolar disorder
D. Alzheimer's disease
389. Which of the following waveforms is most commonly found
when you are awake?
A. Theta
B. Alpha
C. Beta
D. Zeta
390. The REM sleep cycle occur approximately every ____ minutes?
A. 45
B. 60
C. 75
D. 90
391. Which of the following reflexes is not found at birth?
A. Babinski
B. Palmar

C. Moro
D. Flexion
392. Parallel play for toddlers develops in this age range?
A. 5-10 months
B. 10-14 months
C. 12-24 months
D. 24-48 months
393. Which of the following is not a sign of anxiety?
A. Dyspnea
B. Hyperventilation
C. Moist mouth
D. GI symptoms
394. Which of the following best describes a person that is
completely awake falling asleep spontaneously?
A. Cataplexy
B. Narcolepsy
C. Transitional sleep
D. REM absence
395. Which of the following best describes a person that is unable
to tell you were there hand or foot is?
A. Autotopagnosia
B. Cataplexy
C. Ergophobia
D. Anosognosia
396. Which of the following is not a characteristic of a panic
disorder?
A. Nausea
B. Excessive perspiration
C. Urination
D. Chest pain
397. Which of the following categories would a 70 year old adult
be placed in?
A. Intimacy vs. Isolation
B. Generativitiy vs. Stagnation
C. Integrity vs. Despair
D. Longevity vs. Guilt
398. Which of the following categories would a 60 year old adult
be placed in?
A. Intimacy vs. Isolation
B. Generativitiy vs. Stagnation
C. Integrity vs. Despair
D. Longevity vs. Guilt
399. Which of the following categories would a 20 year old adult
be placed in?
A. Intimacy vs. Isolation
B. Generativitiy vs. Stagnation

C. Integrity vs. Despair


D. Longevity vs. Guilt
400. Which of the following describes a person using words that
have no known meaning?
A. Neologisms
B. Neolithic
C. Verbalism
D. Delusional blocking
401. Which of the following is the mRNA start codon in most
cases?
A. UAA
B. AGU
C. AUG
D. UGA
402. Which of the types of RNA is the smallest?
A. mRNA
B. tRNA
C. rRNA
403. Which of the following is not considered a pyrimidine?
A. C
B. T
C. U
D. G
404. Which of the following is paired correctly?
A. A-G
B. C-G
C. A-U
D. G-T
405. Which of the following characterizes a Western blot?
A. Antibody/protein hybridization
B. DNA/RNA combination
C. RNA transcription
D. Polymerase chain reaction
406. Which of the following is the approximate prevalence ratio for
cystic fibrosis?
A. 1: 25,000
B. 1: 5,000
C. 1: 2,000
D. 1: 800
407. Which of the following divisions of cell growth precedes
Mitosis in the cell cycle?
A. G
B. G
C. S
D. G0

408. Down syndrome is directly linked to a genetic abnormality of


chromosome?
A. XXII
B. XXI
C. XIIX
D. XV
409. Which of the following is a characteristic of the HardyWeinberg law?
A. Mating between species occurs at a set rate.
B. Migration is a considerable factor.
C. Mutation occurs at the locus
D. Genotype selection does not occur at the locus
410. Which of the following is not a characteristic of Hurler's
syndrome?
A. Autosomal recessive condition
B. Associated with delayed mental development
C. Spasticity
D. Corneal deficits
411. Which of the following is not a characteristic of Krabbe's
disease?
A. Autosomal recessive condition
B. Spasticity
C. Nausea
D. Optic nerve deficits
412. Which of the following is not a characteristic of Fabry's
disease?
A. X-linked disease
B. Low levels of alpha-galactosidase A
C. Profound muscular weakness
D. Increased levels of ceramide trihexoside
413. Which of the following is not a characteristic of Sickle Cell
Anemia?
A. More common in African Americans
B. Autosomal dominant
C. Mutation in beta-globin
D. Intense chronic pain
414. Which of the following is not a characteristic of the Southern
blot?
A. DNA hybridization
B. Use of a filter and film combination
C. Activated by antigen/antibody reactions
D. Uses a DNA sample
415. Which of the following is not a characteristic of S-adenosylmethionine?
A. May be associated phosphocreatine
B. Considered a rate limiting enzyme of glycolysis

C. Aids in the transfer of methyl


D. Byproduct of Methionine and ATP combination
416. Which of the following is not an activated carrier?
A. ATP
B. SAM
C. TPP
D. GMP
417. The end product of the TCA cycle produces ____ NADH.
A. 3
B. 4
C. 5
D. 6
418. How many ATP are required to transform pyruvate into
glucose?
A. 5
B. 6
C. 7
D. 8
419. Which of the following is not a derivative of the amino acid
(Tryptophan)?
A. Melatonin
B. Serotonin
C. Creatine
D. Niacin
420. . Pompe's disease is a type ___ glycogen storage disease.
f. I
g. II
h. III
i. IV
421. Prothrombin is a ____ globulin and is produced by the _____.
a. Alpha, Kidney
b. Alpha, Liver
c. Beta, Kidney
d. Beta, Liver
422. The right coronary artery divides to form the posterior
interventricular artery and the ___ artery.
A. Marginal
B. LVC
C. RVC
D. LAD
423. Blood flowing into the cardiac veins enters the _______ next.
A. Coronary Sinus
B. Left Ventricle
C. Right Ventricle
D. Left Atrium
424. If you are using a stethoscope and trying to detect the
tricuspid valve which of the following would be the best location?

A. Within 2 inches of the xyphoid process


B. On the right side of the sternum
C. On the left side of the sternum near the midpoint
D. On the left side of the sternum near the midpoint of the sixth
rib
425. Which of the following occurs during ventricular systole?
A. Increased aortic pressure
B. Increased ventricular volume
C. Dup heart sound
D. P wave
426. Which of the following occurs during ventricular diastole?
A. Increased aortic pressure
B. Increased ventricular volume
C. Lub heart sound
D. T wave
427. The innermost layer of a blood vessel is lined with _______
______ cells
A. Simple squamous
B. Stratified squamous
C. Simple cuboidal epithelium
D. Stratified cuboidal epithelium
428. Angiotension can directly cause the release of ____ from the
adrenal cortex.
A. Renin
B. Aldosterone
C. Calcitonin
D. Thyroxine
429. Cardiac output is the product of ____ and ____.
A. HR and Disastolic pressure
B. HR and Stroke Volume
C. HR and EF
D. Diastolic and Systolic pressure
430. Pulmonary edema is most like associated with a failing _____
_____.
A. Right atrium
B. Left atrium
C. Right ventricle
D. Left ventricle
431. Which of the following is the first branch off the aortic arch?
A. Common carotid
B. Brachiocephalic
C. Right Subclavian
D. Thoracic
432. The brachiocephalic artery divides to form the right common
carotid and the ____ ____ artery.
A. Left subclavian

B. Right subclavian
C. Left common carotid
D. Right thoracic artery
433. Which of the following arteries creates the left spenic, hepatic
and gastric arteries?
A. Left sacral artery
B. Celiac artery
C. Suprarenal artery
D. Phrenic artery
434. Which of the following is not considered a major branch off of
the descending thoracic aorta?
A. Mediastinal artery
B. Renal artery
C. Bronchial artery
D. Posterior intercostals artery
435. Which of the following is not considered a major branch off of
the abdominal aorta?
A. Phrenic artery
B. Common iliac artery
C. Gonadal artery
D. Mediastinal artery
436. Which of the following is not considered a major branch off of
the femoral artery?
A. Superficial pudendal arteries
B. Deep external pudendal arteries
C. Superficial circumflex iliac artery
D. Deep circumflex iliac artery
437. Which of the following is not considered a tributary of the
portal vein?
A. Inferior mesenteric vein
B. Splenic vein
C. Left gastric vein
D. Subclavian vein
438. Inside the cranial cavity the vertebral arteries form the ____
artery.
A. Basilar
B. Common Carotid
C. MCA
D. PCA
439. Pulse pressure (pp) is considered the _____.
A. Difference between the systolic and diastolic pressure
B. The sum of the systolic and diastolic pressure
C. The inverse of the blood pressure
D. Half of the systolic pressure
440. _____ nerves can be found joining the SA and AV nodes in the
heart.

A. Accelerator
B. Phrenic
C. Thoracic
D. Gastric
441. Insulin inhibits the release of _______.
A. Glucagon
B. ADH
C. Beta cells
D. Somatostatin
442. Which of the following is caused by insulin release?
A. Increased breakdown of fats
B. Increase breakdown of proteins
C. Decreased blood sugar
D. Causes glucose to be phosphorylated in kidney
443. Glucagon causes increased blood sugar and causes slow
breakdown of glycogen in the liver.
A. TRUE
B. FALSE
444. As blood glucose decreases glucagon is inhibited.
A. TRUE
B. FALSE
445. Glucagon increases blood levels of glucose by causing liver to
breakdown glycogen.
A. TRUE
B. FALSE
446. Which of the following is not true about Type I DM?
A. May be linked to autoimmunity
B. Onset usually prior to age 20
C. Beta islet cells destroyed
D. Does not require insulin injections
447. Which of the following is not true about Type II DM?
A. Considered adult onset diabetes
B. Cause unknown may be due to genetics
C. Require insulin 80% of cases
D. May take a drug that sensitize cells or increase insulin release
448. Which of the following is not an effect of diabetes?
A. Small vessel occlusion
B. Necrosis of extremities

C. Ketone Body production


D. Decreased fat metabolism
449. Which of the following is not an indicator of a hypoglycemic
condition?
A. Fatigue
B. Poor appetite
C. Tachycardia
D. Confusion
450. Which of the following is not an adverse effect of oral
hypoglycemics?
A. Hypoglycemia
B. Headache
C. Rashes
D. Projectile vomiting
451. Which of the following is not an adverse effect of glucagon?
A. Allergic reaction
B. Vomiting
C. Nausea
D. Fever
452. Which of the following drugs may be given as an
immunosuppressant soon after onset of Type I Diabetes?
A. Torsemide
B. Cyclosporine
C. Clofibrate
D. Ceftriaxone
453. Which of the following is not considered an endocrine
hormone?
A. Renin
B. Insulin
C. Glucagon
D. Somatostatin
454. What type of cells secrete glucagon?
A. Beta cells
B. Alpha cells
C. Plasma cells
D. Acinar cells
455. What type of cells secrete insulin?
A. Beta cells
B. Alpha cells

C. Plasma cells
D. Acinar cells
456. Which of the following would not be considered an acute
effect of diabetes mellitus?
A. Polyuria
B. Weight gain
C. Polydipsia
D. Polyphagia
457. Which of the following is not an accurate test for diabetes?
A. Glucose tolerance test
B. HbA
C. Fasting serum glucose
D. Fasting glucagon test
458. Which of the following is not an indicator of diabetic
ketoacidosis?
A. Hyperthermia
B. Nausea/Vomiting
C. Slow and shallow breathing
D. Psychosis leading to dementia
459. Which of the following is not related to a chronic diabetes
mellitus condition?
A. Atherosclerosis
B. Neuropathy
C. Glaucoma
D. Hypotension
460. Which of the following conditions is not linked to diabetic
ketoacidosis?
A. Cerebral edema
B. Arrhythmias
C. Peptic ulcers
D. Mucormycosis
461. Which of the following is not a muscle identified in the rotator
cuff?
A. Teres Major
B. Teres Minor
C. Infraspinatus
D. Supraspinatus
462. Which of the following is not a component of the unhappy
triad?
A. MCL
B. PCL
C. ACL
D. Medial Meniscus
463. Which of the following is not included in the femoral triangle?
A. Femoral Artery

B. Femoral Nerve
C. Femoral Vein
D. Femoral Ligament
464. Which of the following is not a component of the carotid
sheath?
A. Cranial nerve X
B. Common carotid artery
C. Internal jugular vein
D. Cranial nerve IX
465. Which of the following spinal dermatome level corresponds
with the landmark of the inguinal ligament?
A. T10
B. L1
C. L3
D. L5
466. Which of the following nerves innervates the deltoid?
A. Radial
B. Cranial nerve XI
C. Subscapular
D. Axillary
467. Wrist extensors are primarily controlled by what nerve?
A. Radial
B. Ulnar
C. Median
D. Tibial
468. Adductor pollicis in the hand is controlled by which nerve?
A. Radial
B. Ulnar
C. Median
D. Tibial
469. Which of the following arteries is the most frequent site of
coronary artery stenosis?
A. LCA
B. RCA
C. LAD
D. PD
470. Which of the following nerves is not directly linked to the L2L3 spinal level?
A. Tibial
B. Obturator
C. Femoral
471. Which of the following passageways contain the maxillary
nerve and blood vessels?
A. Stylomastoid foramin
B. Inferior orbital fissure

C. Foramen ovale
D. Carotid canal
472. Which of the following passageways contain the facial nerve
and blood vessels?
A. Stylomastoid foramin
B. Inferior orbital fissure
C. Foramen ovale
D. Carotid canal
473. Which of the following passageways contain the internal
carotid artery?
A. Foramen rotundum
B. Condylar canal
C. Foramen ovale
D. Carotid canal
474. Which of the following is the most common site of disc
herniation?
A. C6-7
B. T12-L1
C. L4-5
D. L5-S1
475. Which of the following ligaments is not found in the knee?
A. Patellar ligament
B. Oblique popliteal ligament
C. Arcuate popliteal ligament
D. Deltoid ligament
476. Which of the following nerves innervates the teres minor
muscle?
A. Subscapular nerve
B. Suprascapular nerve
C. Axillary nerve
D. Pectoral nerve
477. Which of the following nerves innervates the pronator teres
muscle?
A. Radial
B. Median
C. Musculocutaneous
D. Ulnar
478. Which of the following supplies the muscles of the perineum?
A. Pudendal nerve
B. Sciatic nerve
C. Femoral nerve
D. Tibial nerve
479. Which of the following eye muscles rotates the eye downward
and away from midline?
A. Inferior oblique
B. Superior oblique

C. Inferior rectus
D. Superior rectus
480. Which of the following eye muscles rotates the eye upward
and toward midline?
A. Inferior oblique
B. Superior oblique
C. Inferior rectus
D. Superior rectus
481. Which of the following is a longitudinal incision through eschar
and down to subcutaneous tissue?
A. Escharotomy
B. Dehiscence
C. Transection
D. Escharotic's procedure
482. Which of the following types of wounds match the criteria:
plantar aspect of foot, met heads, heel?
A. Arterial
B. Plantar
C. Venous
D. Diabetic
483. Which of the following terms matches: water and electrolytes
(clear)?
A. Exudate
B. Transudate
C. Serosanguineous
D. Induration
484. Which of the following edema assessment levels corresponds
with: Depression resolving in 10-15 sec?
A. +1
B. +2
C. +3
D. +4
485. Which of the following terms matches the statement: to
increase the fibrous element; to make hard as in the presence of
cellulites?
A. Induration
B. Necrosis
C. Eschar
D. Maceration
486. Following the rule of nines. What percent would a third degree
burn to the entire arm and back cover?
A. 28%
B. 27%
C. 20%
D. 18%

487. Which of the following matches the defintion: A full thickness


skin loss involving damage or necrosis of subcutaneous tissue
that may extend down to but not through underlying fascia,
infection and/or necrosis may be present?
A. Stage I wound
B. Stage II wound
C. Stage III wound
D. Stage IV wound
488. Which of the following types of wound is indicated by the
definition: relatively painless, decreased with elevation?
A. Arterial
B. Plantar
C. Venous
D. Diabetic
489. Which of the following matches the definition: The loss of
circulatory fluids into interstitial spaces?
A. Hypovolemia
B. Necrosis
C. Eschar
D. Maceration
490. An emollient has a/an _____ effect.
A. Pruritic
B. Antipruritic
C. Rupture
D. Impetigo
491. Which of the following is the outermost layer of the
epidermis?
A. Stratum spinosum
B. Stratum corneum
C. Stratum granulosum
D. Stratum basale
492. Which of the following is the deepest layer of the epidermis?
A. Stratum spinosum
B. Stratum corneum
C. Stratum granulosum
D. Stratum basale
493. Which of the following is beneath the stratum corneum?
A. Stratum spinosum
B. Stratum corneum
C. Stratum granulosum
D. Stratum basale
494. Vitamin D is created from _________ by skin cells.
A. Dehydrocholesterol
B. Cholesterol
C. Hydrocholesterol
D. Hydrodermis

495. Which of the following is another name for blackheads


associated with acme?
A. Pustules
B. Sebaceous
C. Eccrine
D. Comedones
496. Which of th following identifies skin from a cadaver used in a
burn graft?
A. Homograft
B. Autograft
C. Allograft
D. Xenograft
497. Which of the following is a disease characterized by
hyperactive sebaceous glands and often associated with
dandruff?
A. Keloid
B. Seborrhea
C. Eczema
D. Urticaria
498. Which of the following is a disease characterized by the
presence of hives?
A. Keloid
B. Seborrhea
C. Eczema
D. Urticaria
499. Which of the following is a disease characterized by a skin
rash that is blistering and itchy?
A. Keloid
B. Seborrhea
C. Eczema
D. Urticaria
500. Sebaceous glands secrete _______.
A. Sebum
B. Impetigo
C. Serous
D. Sirius

You might also like